IMPORTANT MESSAGE FOR USERS OF THIS SUMMARY:

Please note that this summary was scanned in from a printed copy (with handwritten notes on some pages), without ANY corrections made to the text recognition. Therefore, much of the text has odd typos and extraneous characters from errors in the text recognition.

Caveat lector!

Pubdocs Online Summaries Library [email protected] POLICE POWERS

1. SEARCH AND SEIZURE Colleen Liggett (499-0496)

(Casebook 5 7-76, 94-98, 99-112, 122- 1 3 ) 7) Common Law Limitations to the Power to Search and Seize Ghani v. Jones (57) -- English Court of Appeal 1970 Synopsis: For the power to search and seize to be valid at common law, the police must comply with 5 conditions: (1) reasonable grounds for believing that a serious offence has been committed; (2) reasonable grounds for believing that the article in question is either the fruit of the crime or is the instrument by which the crime was committed or is. material evidence to prove the commission of the crime; (3) reasonable grounds for believing that the person in possession of it has himself committed the crime, or is implicated in it, or is accessory to it, or at any rate his refusal must be quite unreasonable; (4) police must not keep the article, nor prevent its removal, for any longer than is reasonably necessary to complete their investigations or preserve it for evidence (make copy if possible and return original); and (5) lawfulness of conduct of police must be judged at the time, and not by what happens afterwards

Facts: - wife came from Pakistan to join her husband in England at his parent's house - wife disappeared in November 1968 not to be seen since - husband returned to Pakistan in April 1969, and husband's sister came to England to stay with her parents - police believed wife had been murdered - in June 1969, police went to husband's parents house to make inquiries, having no search warrant - father asked police in: I officer questioned him while the other searched the house - police requested their passports, so the father gave his own, his wife's, and his daughter's (husband's sister's) which the police took away, in addition to letters received from other members of the family - in July 1969, father instructed solicitors to request the return of the passports and letters, bLit the police refused, so the family began this action - police gave affidavit indicating that the documents will be of evidential value, and that the family can assist them in their inquiries, so they fear that if the family leaves the country (for which they need the passports), they will not return no allegation in the affidavit that the family was implicated in the suspected murder Issue: is the retention of the passports and letters a valid exercise of police power? i.e. is it sufficient justification in law -that the police believe the documents to be of "evidential value" on a prosecution for murder? Held: unlawful seizure, so mandatory order to return the items Lord Denning M.R.: - police seized property without warrant, without making an arrest., and retained it without consent - search with warrant, or arrest with or without a warrant: police are entitled to take any goods they find in his possession or in his house which they reasonably believe to be material evidence in relation to the crime for which he is arrested or for which they enter; may take other goods they come upon in the course of their search which show him to be implicated in some other crime provided they act reasonably and detain them no longer than is necessary - police should be able to do whatever is necessary and reasonable to preserve evidence of the crime - applying the above 5 conditions to this case, the police meet (1), but fail (2), (3), and (4) - the real reason why the passports have not been returned is because the officers wish to prevent the plaintiffs from leaving this country pending policy inquiries, which is not a legitimate ground for holding them - a person's liberty of movement -must not be taken away on a suspicion which is not grave enough to warrant his arrest

Statutory Power of Search and Seizure _sS 461 --~ 89

Laporte v. Laganiere (64) -- Quebec Queen's Bench 1972

Synopsis: The authorization to search and seize must be conferred by statute or at common law.

Facts: warrant purporting to authorize a search in the -body of the petitioner Roger Laporte for bullets alleged to have been'fired from police 'revolvers in an exchange of gunfire, during a hold-up in Knowlton a year and a half ago - Laporte was arrested on another matter, but the police have reason to suspect he was involved in the hold-up in Knowlton - Laporte has scars on neck and shoulder resembling bullet wounds and X-rays reveal foreign body corresponding in size and shape to a 39-calibre slug - to remove the bullet requires general anaesthetic, and so a certain element of risk - normally, operation would not be performed without consent, which Laporte has not given here Issue: did the justice have jurisdiction to issue the search warrant? Held: no jurisdiction to issue the search warrant Hugessen J.: - is a living human body a "building, receptacle or place" (s. 487) into which surgical intrusion may be justified by means of a search warrant? - the body is not a building nor a receptacle (which cannot include the interior of a living human body) nor a place (a geographic and not an anatomical location, which was not intended to include the interior of a human body)

-Numt 5.)%, (&,CLYCh warr,-Wyt bictrL 5cz-Mpir-) ') SS, 4S1, 0A - 01, 09 (witch iAjwcLM bcdA1 5~A65iance5 fcj DNA cawlis~ls~. - if we authorize police today to probe into a man's shoulder for evidence against him, what is to prevent them tomorrow from opening his brain or other vital organs for the same purpose? - balance protection of society with protection of rights of individuals - if Crown cannot prove its case against Laporte without doing physical violence to his person then it is better that the case be not proved - justice had no jurisdiction, either by statute or at common law, to issue this warrant - even if the operation pro2osed were minor, would not sanction it fJ12 VC1 ~_AOA LX ffC.M ~ r4b, S"ffvwl de-Kils~ A (lei' s, 4S1 M, 3 recjAi-xYwrA5 -16,,lid scc t %) i -,m*OA (APOVI CCM1 W' 'th

0)~%A_Sin,U envi6t C-C I tj%~diwlf q_ 'A ~ndef0d& asYcsgmerd c4 foz-15 avid in J iZjZ~,fi'CWJ cts -lo "ie4W-r'Jp i'ss"c OL -SCOITh Wcv-tranf 4 o6licLivl S4RClem eiphw of ob~ P-efs it, relctiilkn 10 e L*!~ 51', CL M 0#6xe, Re Gillis and R. (69) -- Quebec Superior Court 1982'

Synopsis: To be valid, a search warrant must describe the objects sought with sufficient precision with respect to their category, and with respect to their relation to the offence for which they are to provide evidence.

Facts: - warrant purporting to authorize search for financial statements, etc. in connection with a fraud Issue: is the search warrant sufficiently precise so that the issuing justice will not be held to have exceeded his jurisdiction? Held: no jurisdiction to issue the search warrant Boilard J.: issuance of a search warrant is a judicial act whereby justice must decide whether there are -reasonable grounds to believe that the objects described in the information are located in the place indicated by the informant and whether these objects will provide evidence of the commission of the offence set out in the information (i.e. existence and presence of certain objects in a given place, and their probative value) - purpose of search warrant is to secure things that will in themselves be relevant to a case to be proved, not to become a fishing trip - the justice cannot leave the ultimate decision as to the "reasonable grounds" to the discretion of the informant - where the connection between the items sought and the offence is one that might be gathered easily by inference from the very nature of the offence and the material sought, the informant is not obliged to underline the obvious - the objects or documents sought under the search warrant must be described with sufficient precision, not only with respect to their category, but also with respect to their relation to the offence for which they are to provide evidence - specify which financial statements one is looking for; specify relation between the fraud and the documents sought - search warrant must set out with sufficient precision the nature and identity of the documents sought so that whoever conducts the search, and also whoever is subject to the search, can, upon reading the warrant, know what is being sought and what may be seized - not sufficient to state the documents sought concern a fraud; they must be more adequately identified by mentioning their date or other characteristics such as numbers, amounts, in short, some indication which ties the document to the offence for which the warrant was issued - search warrant must be quashed for excess of jurisdiction in not adequately detailing the objects C) sought in the search warrant - documents seized must be returned - police cannot exercise discretion over which documents they wish to retain

Common Law Power of Search and Seizure

R. v. Brezack (94) -- Ontario Court of Appeal 1949

Synopsis: There is a valid power of search incident to arrest, provided there are reasonable grounds for the arrest.

Facts: - charge of unlawful assault of peace officer while engaged in the execution of his duty of making an arrest - police had information leading them to believe A had narcotics in his possession, to be found concealed in his mouth - A had previous conviction for possession of narcotics - police, while observing the Golden Grill (GG), saw A proceed towards GG, and others known to be drug addicts proceed toward GG police rushed on A before he entered GG, one (X) seizing him by the arms, the other (Y) catchlill" -him by the throat to prevent him swallowing anything he' might have in his mouth - all 3 fell to the ground and a struggle ensued - Y tried to insert his finger in A's mouth to recover the drug he assumed was there, and each time A bit his finger - finally, Y succeeded in getting his finger in A's mouth, to find no drug there - police then searched A's clothing, but did not find any drugs on him - police then took A to his car around the corner and searched the car, to find in various places on the floor 5 capsules containing narcotics - A was taken into custody, and later this charge of assaulting the police was laid, the assault being said to constitute the biting of Y's finger and the striking and kicking in the struggle on the ground Issue: whether the search was valid Held: search is valid as incident to arrest Robertson C.10.: - Y was engaged in lawful execution of his duty as a peace officer making an arrest, and believing the information he had, was entitled to search in A's mouth for evidence of the offence of which lie believed A to be guilty - Y was warranted in arresting A on the information he had, although his information was wrong as to where the drug would be found - Y's duty, in making the arrest, was to make reasonable efforts to obtain possession of any narcotilc that he believed to be illegally in A's possession, both for purpose of using it as evidence of possession, and to prevent A disposing of the drug

I I - the finding of the narcotic in A's car a few minutes later strongly supports the reasonableness of the constable's belief in the information he had received that the prohibited drug would be found concealed in A's mouth - the search made is justifiable as an incident of the arrest - it is sufficient if the circumstances are such as to justify the search as a reasonable precaution - there was an arrest here when the police seized the person of the A -- police should be instructed that there are limits upon their right of search, including search of the person, but they are not to be encumbered by technicalities in handling the situations with which they often have to deal in narcotic cases, which permit them little time for deliberation and require the stern exercise of such rights of search as they possess

Laporte v. Laganiere (96) -- Quebec Queen's Bench 1972

Synopsis: There is a common law right to search incident to arrest in order to make the arrest effective, to ensure that evidence does not disappear, and to prevent the commission of a further offence.

Facts: - see above (extending power to search incident to an arrest to include a surgical operation) Issue: whether there is a common law right to search incident to arrest Held: there is, but there are limits attached Hugesson J.: - there is a common law right to search a prisoner at the time of his arrest - the common law right of search is limited to that search which is incidental to the making of an arrest or the continued detention of the prisoner in safe custody: to make the arrest effective, to ensure that evidence does not disappear, and to prevent the commission of a further offence - there's nothing in the cases which would justify a surgical intrusion into the body of a prisoner many months after his arrest, for the purpose of obtaining evidence against him on a charge other than that for which he is being held

R. v. Tomaso (97) -- Ontario Court of Appeal 1989

Synopsis: There is a common law power to search incident to arrest. The search may precede the arrest provided that the police have reasonable grounds, prior to the search, for arresting the accused.

Facts: - charge of dangerous driving resulting in fatality - while unconscious in hospital, A's blood from his bleeding ear was collected by police - analysis showed A was impaired at time of accident - A was charged 2 weeks after blood sample was obtained

I Issue: whether the seizure of the blood was reasonable due to its timing Held: seizure was unreasonable Howland C.J.O.: - there is a common law power to search as incident to a lawful arrest - the search preceding arrest may be incident to arrest provided that the police had reasonable grounds, prior to the search, for arresting A - at time of sample, police had reasonable and probable grounds for believing that A was impaired, but did not have reasonable grounds to arrest A on charge of criminal negligence - blood analysis was part of justification for arrest on charge of criminal negligence - cannot justify seizure on basis that it was incident to arrest 2 weeks later for offences with which A was charged - amendments to Criminal Code in s. 254(3) and s. 256 came into force too late to be applicable to this case

Powers of Search Incidental to Police Duties

Dedman v. R. (100, 106) -- Supreme Court of Canada 1985

Synopsis: The authority for police action must be conferred by statute or derive from common law. The test to determine whether the police had authority for their actions is: (1) whether the police conducts falls within the scope of a duty imposed by statute or recognized at common law (i.e. were the police performing a duty prescribed by statute or common law?); and (2) whether such conduct involved an unjustifiable use of powers associated with that duty (i.e. was the power exercised reasonably?)

Facts: - charge of failing, without reasonable excuse, to provide a breath sample - A had been apprehended as part of RIDE program (random vehicle stop program) Issue: whether random stop by police of car as part of RIDE program is unlawful as being made without statutory or common law authority, despite A's compliance with signal to stop Held: random stop is lawful as being made under police's common law duty to prevent crime and protect life and property LeDain J. (Majority): - police only act lawfully if their authority is conferred by statute or derived from their common law duties - intimidating nature of police action and uncertainty as to extent of police powers means that compliance in such circumstances cannot be regarded as voluntary; possible criminal liability for failure to comply constitutes effective compulsion or coercion - at common law the principal duties of police officers are the preservation of the peace, the prevention of crime, and the protection of life and property, from which is derived the duty to control traffic on the public roads follows test in Waterfield to determine whether the police officer had authority for what he did

Weie Dtcivvx)~n alst~ VO cis * ct+ WASO-4 -1o a se o, tch m USi have- be e n ( e cd Q nd vc! kAnfO-r-,J, 4+)C4 Pusc V1 Se cki(ch e d YOU si KAVC -tv b, peo~cc-~ecl ckjctinsi unveciscocx6it sectvck anct 5A,6EAA cv)cLveAez -jo i)ckvc- ,jctivekA -Me c-cos-640ibexti C13, I -)sc q - , c, -zjuat t4~e j)LAiv,6Aa1 ryNuSi be &LA;cvC c4 pc+en~'al c0i , veiTes (see above) RIDE not authorized by statute the right to circulate in a motor vehicle on the public highway is not a fundamental liberty like the ordinary right of movement of the individual, but a licensed activity that is subject to regulation and control for the protection of life and property - random stop under RIDE falls within general scope of duties of police to prevent crime and to protect life and property by the control of traffic, and so is authority granted at common law, thereby meeting step (1) of the Waterfield test - notion of reasonably necessary in step (2): the interference with liberty must be necessary for the carrying out of the particular police duty and it must be reasonable, having regard to the nature of the liberty interfered with and the importance of the public purpose served by the interference - having regard to the importance of the public purpose served (RIDE deters driving) t e random stop, as a police action necessary to the carrying out of that purpose, was not an unreasonable interference with the right to circulate on the public highway, thereby meeting step (2) of the Waterfield test as not being an unjustifiable use of power associated with the police duty - there was common law authority for the random vehicle stop for the purpose contemplated by the RIDE program Dickson C.J. (Dissent): - agrees that police only act lawfully when they exercise authority conferred by statute or at common law - agrees that submission to police exercise of apparent authority cannot be characterized as voluntary or consensual, unless it was clear to the person at the time that he was free to refuse to comply - agrees that RIDE fiot authorized by statute - police, in carrying out their general duties as law enforcement officers of the State, have limited powers and are only entitled to interfere with the liberty or property of the citizen to the extent authorized by law - agrees with LeDain J.'s formulation of the common law duties of police - action to detect and prevent people from driving while impaired by alcohol, an offence under the Criminal Code, falls within the general duties of the police, but it is necessary to distinguish the duties of police from the power, or lawful authority, they possess to execute those duties - police have no lawful authority to execute their general duties by means of random stops of motorists when they have no reason to believe, prior to the stop, that the motorist has committed, is committing or will commit a criminal offence - the Waterfleld test provides no support for police conduct, where the conduct is unlawful at common law - police are not entitled to execute their duty by unlawful means - to find that arbitrary police action is justified simply because it is directed at the fulfillment of police duties would be to sanction a dangerous exception to the supremacy of law - it is the function of the Legislature, not the courts, to authorize arbitrary police action that would otherwise be unlawful as a violation of rights traditionally protected at common law - since police lack legal authority to detain a person for questioning or for purposes of investigation at common law, even on suspicion, short of arrest, there's no basis for the power to stop and detain a motorist in the circumstances of this case - RIDE involves the police stopping motorists on an entirely arbitrary basis to question them

! - - - T - -1 1- -1-- - - - here, police had no grounds to reasonably suspect that A had committed, was committing or was about to commit a criminal offence before he was requested to stop - police stopped and detained A arbitrarily to investigate whether he might be committing a criminal offence - the fact that driving a motor vehicle is a licensed activity subject to regulation and control in the interests of safety is irrelevant to police power if the conditions for licensing have been met and are adhered to; the curtailment of liberty by the police in no way flows from the fact of licensing or any other regulation - without validly enacted legislation to support them, the random stops by the police under the RIDE program are unlawful

Colet v. R. (102) -- Supreme Court of Canada 1981

Synopsis: The right to seize does not include the right to search. The right to enter private property and search must be expressly authorized.

Facts: - charge of 2 counts of attempted murder, and 2 counts of intending to cause bodily harm arising OUt of A's defence of his property against what he deemed to be the wrongful intrusion of police officer acting under purported authority of warrant to seize - Public Works were to clean up A's property, but A was very incensed at this prospect, making it known that he would defend his property by all possible means - RCMP feared A might use firearms or other offensive weapons to protect his property - RCMP obtained warrant pursuant to Criminal Code authorizing seizure of firearms or other offensive weapons in A's possession - A felt police had no authority to search his home, and so mounted the roof, from which he threw some gasoline at the police Issue: whether the property rights of an individual can be invaded otherwise than with specific statutory authority Held: no, so a warrant authorizing seizure is not sufficient to authorize search Ritchie J.: - broad basic principle of sanctity of the home is subject to the exception that upon proper demand police may enter property against an individual's will in search of a fugitive from justice whose arrest they consider to be justified - the right to seize does not include the right to search - police are not justified in making an entry unless they have first announced their presence and demonstrated their authority by stating a lawful reason for their entry - warrant here was not a warrant to search the premises, nor did the authority to seize specified in the warrant carry with it the right to enter and search - any provisions authorizing police officers to search and enter private property must be phrased in express terms - if Parliament intended to include the power to search in the provisions of the Criminal Code, the failure to do so was a clear case of legislative oversight, but that power which has not been

1~)c &,Oey)ij)cxJ Code has been &M", CCL s~inet- C01ei ---b react "se-oah -~( and &,-ae " b;. 16 4iie ~,Iec,'Sc r0in i,1 Colf i is mcc4. expressly conferred cannot be supplied by invoking the provisions of the Interpretation Act, which provides that all powers necessary to enable a person to do his job shall be deemed to be also given where power is given to do a job - police were acting without authority in attempting to enter and search A's property, and they were therefore trespassers

Constitutional Limits on the Power to Search and Seize

Hunter v. Southam Inc. (122) -- Supreme Court of Canada 1984

Synopsis: To be consistent with s. 8 of the Charter's guarantee against unreasonable search and seizure: (1) there must be prior authorization under the law for the search and seizure; (2) the law authorizing the search and seizure must be reasonable (it is primafacie unreasonable to search and seize without a warrant); (3) the manner in which the power under the law is exercised, i.e. in which the search and seizure is carried out, must be reasonable

Facts: - s. 10 of the Combines Investigation Act permits Restrictive Trade Practices Commission (RTPC) member to approve of the Director of Investigation's authorizing a combines investigator to enter premises, search and seize any document that he considers may afford evidence - RTPC member Roseman approved of appellant Hunter's, as Director of Investigation, authorization of combines investigators to enter and examine documents at business premises of Edmonton Journal, a division of the respondent corporation Southam Inc. - both Hunter's authorization and Roseman's approval occurred just days before the proclamation of the Charter, of which is relevant s. 8's guarantee against unreasonable search and seizure - the actual search took place after the Charter was proclaimed - the authorization is broad, allowing investigators to enter premises of Southam at the stated address "and elsewhere in Canada" - investigators requested to see every file, declined to say who had initiated the inquiry, or under which section of the Act the inquiry had been begun, or to give more specific information as to the subject matter of the inquiry than that contained in the authorization to search - Southam filed for interlocutory injunction to stop the search, questioning whether the search was in violation of s. 8 of the Charter, which motion was denied, the judge feeling that the balance of convenience militated in favour of denying the interlocutory injunction pending trial of the matter Issue: whether s. 10 of the Combines Investigation Act is inconsistent with s. 8 of the Charter by reason of authorizing unreasonable searches and seizures Held: s. 10 of the Combines Investigation Act violates s. 8 of the Charter, so is of no force and effect Dickson J.: - appeal court listed procedural safeguards required by the common law for reasonable searches and seizures: (1) the power to authorize a search and seizure must be given to an impartial and independent person who is bound to act judicially in discharging that function; (2) evidence must satisfy the justice that the person seeking the authority has reasonable grounds to suspect that an offence has been committed; (3) evidence must satisfy the justice that the person seeking the authority has reasonable grounds to believe that something that will afford evidence may be recovered - the conduct of the appellants is not in issue, but rather the legislation under which they acted - s. 8 guarantee is vague and open - need for broad perspective in approaching constitutional documents; give the constitution a large and liberal interpretation, a generous interpretation; broad, purposive analysis, interpreting specific provisions of a constitutional document in the light of its larger objects - purpose of Charter is to guarantee and to protect, within the limits of reason, the enjoyment of the rights and freedoms it enshrines - an assessment of the constitutionality of a search and seizure, or of a statute authorizing a search or seizure, must focus on its "reasonable" or "unreasonable" impact on the subject of the search or the seizure, and not simply on its rationality in furthering some valid government objective - purpose of s. 8 is not restricted to the protection of property or associated with the law of trespass - s. 8 protects a reasonable expectation of privacy: an assessment must be made as to whether in a particular situation the public's interest in being left alone by government must give way to the government's interest in intruding on the individual's privacy in order to advance its goals, notablv those of law enforcement - requires system of prior authorization, not one of subsequent validation, because s. 8 requires a means ofpreventing unjustified searches before they happen, not simply of determining, after the fact, whether they ought to have occurred in the first place requirement of warrant puts onus on state to demonstrate the superiority of its interest to that of the individual, according with apparent intention of Charter to prefer, where feasible, the right of the individual to be free from state interference to the interests of the state in advancing its purposes through such interference - where it is feasible to obtain prior authorization, such authorization is a precondition for a valid search and seizure - require the party seeking to justify a warrantless search to rebut the presumption of unreasonableness - in absence of valid procedure for prior authorization, searches conducted under the Combines Investigation Act would be unreasonable - for authorization procedure to be meaningful, the person authorizing the search must be able to assess the evidence in an entirely neutral and impartial manner; the person performing the function of authorizing a search need not be a judge, but he must at a minimum be capable of acting judicially - Director under CIA exercises investigatory rather than adjudicatory powers - RTPC member also has investigatory powers, so that vitiates his ability to act in a judicial capacitV when authorizing a search: administrative nature of the commission's investigatory duties ill accords with the neutrality and detachment necessary to assess whether the evidence reveals that the point has been reached where the interests of the individual must constitutionally give way to those of the state - prior authorization procedure under CIA is inadequate to satisfy the requirements of s. 8 of the Charter, so search carried out under that authority is unreasonable - will still consider whether, even if prior authorization procedure in CIA did speci fy a neutral and detached arbiter to authorize searches, it would nevertheless remain inconsistent with s. 8

I I - I Tl- - s. 10 CIA mentions only 2 criteria for granting authorization: (1) an inquiry under the Act must be in progress; and (2) the Director must believe that the premises may contain relevant evidence decision of Director to exercise powers of search and seizure is effectively unreviewable, so that the extent of the privacy of the individual would be left to the discretion of the director, making the provision authorizing such an unreviewable power inconsistent with s. 8 - it's inadequate to read s. 10 as simply allowing the authorizing party to satisfy on these questions,, without requiring him to do so - it's inadequate to require the authorizing party to satisfy himself as to the legality of the inquiry and the reasonableness of the Director's belief in the existence of relevant evidence, because of the stipulation of a reasonable belief that evidence may be uncovered in the search - state's interest in detecting and preventing crime begins to prevail over the individual's interest In being left alone at the point where credibly-based probability replaces suspicion - minimum standard consistent with s. 8 for authorizing search and seizure: reasonable and probable grounds, established upon oath, to believe that an offence has been committed and that there is evidence to be found at the pace of the search - s. 10 is inconsistent with this standard, and so is inconsistent with s 8 - refuses to read in the appropriate standard into s. 10 rather than striking it down - it is the legislature's responsibility to enact legislation that embodies appropriate safeguards to comply with the Constitution's requirements - s. 10 is of no force and effect - appellants made no submissions under s. I of the Charter, so unnecessary to discuss it ISEARCH AND SEIZURE: Warrantless Searche

R. v. [1987] S.C.R. p. 143 facts: police officers were conducting surveillance of village pub in connectn w heroin investigation; they observed acc and her husb at one of tables; the acc's husb left pub and police followed him and searched car and found heroin; police returned to pub and one of them grabbed acc by throat to prevent her from swallowing any evidence that may be in her mouth; during the struggle, the officer observed something in her hand, told her to drop it, and retrieved a greeen balloon, which contained heroin

- i) was this throat search unreasonable bec it was not carried out within s. 10 of Narcotic Control Act ii) was this search unreasonable bec, while being within s. 10, the officer carried out the search in a manner that made the search unreasonable? held: appeal allowed and new trial ordered bec Crown was not able to properly establish reasonable grounds of police officer due to objection made by appelants

Eafiw Lamer ==> In Hunter case, SCC held that US formulation regarding warantless searches (=warantless search is prima facie "unreasonable") was applicable to the concept of "unreasonableness" under s.8 and further held that the party seeking to justify a warantless search is required to rebut this presumption of unreasonableness.

"This shifts the burden of persuasion from the appelant to the Crown. As a result once the app has demonstrated that search was a warantless one, the Crown has the burden of showing that search was, on a balance of probabilities, reasonable. A search will be reas if it is authorized by law, if the law itself is reasonable and if the manner in which the search was carried out is reasonable. In this case, Crown argued that the search was carried out under s. 10 (1) of Narc Control Act.. For the search to be lawful under s. 10, the Crown must establish that the officer believed on reasonable grounds that there wasa narcotic in the place where the person searched was found. The nature of the belief will also determine whether the manner in which the search was carried out was reas..Without very specific information, a seizure by the throat, as in this case, would be unreas. Of course, if he is lawfully searching a person whom he believes on reas gmds to be a "drug handler" then the "throathold" would not be unreas.

Bec of presumptn of unreasonableness, the Crown in this case had to present evidence of the officer's belief and the reas grounds for that behef. However, the Crown failed to establish such reas grnds, in the examinatri of constable and when it attempted to do so in re-examination, the appelant's counsel objected. As a result, the Crown never did establish the constable's reas grounds. Without such evidence, it is clear that the trial judge was correct in concluding that the search was unreas bec it was unlawful and carried out with unnecessary violence."

But -- there was problem with appelant's objection and so new trial was ordered. case is nevertheless key bec of test set out above by Lamer R. v. LD.D. (1987) Sask C.A. p. 145 fKk; officer stopped car (bec he saw it attempting to re-enter the highway from a snowy ditch) in which respondent (14 yr old girl) was travelling with her two older brothers; officer said he smelled liquor on the driver and detected a strong smell of liquor in the car; respondent and driver gave evidence that no liquor had been consumed in the car and that there was no liquor in the car; officer searched the car for open liquor; respondent was alone in the backseat; officer picked up her purse, which was on the seat beside her, felt it, unzipped it, and found seven 35-mm film containers containing hash oil which, when sold on the street, was worth betwn $7000 and $10000; officer found one empty beer bottle under the front seat but there is no indicatn that the bottle had contained liquor recently; officer arrested driver and searched him and he then arrested the other two; girl was charged with possession of canabis resin for the purpose of trafficking under s.4(2) of Narc Control Act

L%= was search of girl's purse unreas? bd& appeal allowed and new trial ordered since trial judge did not address s.24(2) (illegally obtained evidence) and bec: trial judge made no findings of fact on a number of matters which might be relevant to an application of principles in Collins (which had not been decided at time of trial) re: whether manner of search was reas

MAX Sherstobitoff J ~> "The point is.. that the manner and scope of the search must bear some relationship to the offence suspected and the evidence sought. The suspected offence here is of a minor nature ... That raises this ?: does suspicion of such a minor offence wan-ant a search of the intrusive nature permitted by s.131 (=officer may without warrant search for the liquor wherever he may suspect it to be, and may, if need be by force, search the person himself and seize and remove any liquor found and the packages in which it is kept) Here the purse of the passenger was searched and the defence questioned whether the purse had the capacity to hold a liquor bottle and inferentially, whether the search was really for drugs rather than liquor. Since the judge made no findings of fact, the matter cannot be decided here. SCC in Hunter cautioned agnst "fishing expeditions." In Collins, the court said that the manner of the search must be reasonable. It is to be taken from those cases that the manner and scope of the search must bear some relationship to the offense suspected and the evidence relating to that offense which is sought.

SCC considered the application of s.8 of the Charter in Hunter and concluded that warrantless searches were prima facie unreasonable. However, the court did recognize that there were situations where it would not be feasible to obtain a prior authorization and that it might in such cases, be reas to search without a warrant..

The Canadian cases above referred to have examined carefully the constitutional right in question, privacy, ad have noted that there is a lesser expectation of that right in a vehicle as opposed to a dwelling house or a place of business. There can be no doubt that the exception to the warrant requirement referred to in Hunter will often apply to searches of vehicles when there are exigent circumstances which make it impractical to obtain a warrant. However, there is no basis in s.8 or in Hunter to found a blanket exgMt&on for vehicle searches. The limits of the exception will be delineated on a case-by-case basis bec they will depend on the facts of each case.

However, the onus will, in each case of a warrantless search, be upon the Crown to establish that a warrantless search was reas. The minimum requirements will be: i) that the vehicle be stopped or the occupants be detained lawfully ii) that the officer conducting the search have reas and prob grnds to believe that an offense has been, is being or is about to be committed and that a search will disclose eivdence relevant to that offense iii) that exigent circumstances, such as imminent loss, removal or destruction of the evidence make it not feasible to obtain a warrant iv) that the scope of the search itself bear a reasonable relationship to the offense suspected and the evidence sought.

S. 131 of the Liquor Act, insofar as it permits searches which go beyond the exception, contravenes s.8 of the Charter and is, to that extent, unenforceable. R v. Dyment [1988] S.C.R. p. 149 facts: a doctor found the accused sitting in the driver's seatof his vehicle in a bloody condition; police officer drove the acc to hospital but did not arrest or detain him; neither officer nor doctor noted any signs that the acc had been drinking; the doc, when preparing to suture the acc's head, collected a sample of free-flowing blood for medical purposes; the acc did not consent to, and was not even aware of, the sample being taken, as he was concussed; after taking the sample, the doc gave it to the officer; at that time, there was no Crim Code provision requiring a person to give a sample of blood; officer had not requested such a sample and did not have a search warrant

1MM was this search unreas and shld evidence therefore be excluded under s.24(2) (illegally obtained evid)? bgl(L appeal dismissed --> evidence shld be excluded under s. 24(2) (illegally obtained evidence) (McIntyre J. dissented on the basis that admission of the evidence wld not bring the administration of justice into disrepute)

Lame =--> "The fact that the doctor, at the time he remitted the vial to the police, had in his possession the respondent's blood subject to a duty to respect the respondenfs privacy is sufficient to qualify the police's receiving of the vial of blood without the consent of the doctor's patient as being a "seizure" as that term is meant in s.8 of the Charter. As it was a warrantless seizure, the evidentiary burden of establishing that it was nevertheless a lawful seizure rested upon the shoulders of the Crown (Hunter) .... The Crown has not adduced any evidence supportive of that conclusion, and there is no evidence on record in that regard. The seizure was therefore an unlwful one. Furthermore, there was no urgency or any other reason justifying such a seizure without first obtaining a warrant, assuming, of course, that the police officer, after having spoken to the doctor, cld bring himself within the conditions required for obtaining a valid warrant..

[he then cites Collins: a search will be reasonable if it is authorized by law, if the law itself is reas and the manner in which the search was carried out is reasonable.]

The fact that the seizure in this case was unlawful, in my view, ends the inquiry as to whether the search was an ureas one. The only issue left is whether the evid shld be excluded. For the reasons given by my brother LaForest, I am of the view that this is a case where the evidence must be excluded."

LaForest ==> "In this case... there was no search. The doctor simply collected the blood as it flowed from an open wound and it was later handed over by him to the police officer. It should be observed, however, that s.8 of the Charter does not protect only against searches, or against seizures made in connection with searches. It protects against searches or seizures ... M I see it, the essence of a seizure under s.8 is the taking of a thing from a person by a public authority without that person's consent .. [W]hen the officer took the sample from the doctor he took something that the doctor held for medical purposes only, subject to a well-founded expectation that it was to be kept private. In the circumstances in which it was taken, Mr. Dyment was entitled to a reasonable expectaw of privacy .... S.8 is concerned not merely with the protection of property but with the protection of the privacy interests of individuals from search or seizure.... In the present case, the respondent may ... be deemed to have impliedly consented to a sample being taken for medical purposes but he retained an expectation that his privacy interestin the sample would continue after the time of its taking ... [Tlhe sample was surrounded by an aura of privacy meriting Charter protection. For ft - state to take it in violation of a pLtienVs right to pri Lvaa constitutes a seizure for the p-uW9-,g-s of s.8.

BUT: The seizure here was not reasonable. To begin with, I do not accept that the violation of Mr. Dyment's privacy interests was minimal ... The seizure infringed upon all the spheres of privacy already identified, spatial, personal and informational. Under these circumstances, it seems to me that, in the absence of pressing necessity, it was unreasonable for the police officer to act as he did .... The taking by the police of a blood sample from a doctor who has obtained it for medical purposes cannot be viewed as anything but unreasonable in the absence of compelling circumstances of pressing necessity. The need to follow established rules in cases like this is overwhelming...

In Hunter, this court expressed the view that "where it is feasible to obtain prior authorization such authorization is a preconditio for a constitutionally valid search and seizure .....The focus here is not predominantly on the taking of the sample by the physician, but on its seizure by the police. It seems obvious to me that, assuming adequate cause, the police officer could have obtained a warrant authorizing the seizure of the sample and there was nothing that could justify his failure to do so. No exigent circumstances existed, bec the blood had been extracted within the 2 hour limit that is said to be essential to an accurate test. Indeed, when the facts are scrutinized, the most probable reason why no warrant was obtained was because the officer lacked the requisite belief that the accused had committed an offense and that the seizure 'was likely to yield evidence which was probative of that offence. ... Quite simply, the constit does not tolerate a "low standard which would validate intrusion on the basis of suspicion and authorize fishing expeditions of considerable latitude" (Hunter): if anything, when the search and seizure relates to the integrity of the body rather than the home, for ex, the standard is even higher than usual. I have no difficulty in concluding that, even if the seizure in this case was legal, it was cleully unreasonable in terms QLAA.-

,The Charter breach, in my view, was a very serious one. In Pohoretsky, Lamer J. observed that 'a violation of the sanctity of a person's body is much more serious than that of his office or his home." --- LaForest concluded that evidence shld therefore be exlcuded. dissent McIntyre =--> evidence shld not be excluded bec its admission w1d not bring the administration of justice into disrepute. Cloutier v. LangLois [1990] S.C.R. p. 154

1q= the accused (=both officers of MUC police dept) stopped a vehicle which they had seen violating a municipal by-law; they arrested the driver (the respondent) upon being informed by police headquarters that a warrant of commital had been issued against him in the Municipal Ct. for unpaid traffic fmes; there was evidence that the respondent's conduct, in the wrds of Court of Appeal judges, "unpleasant" and "highly agitated and verbally abusive"; when the officers asked the respondent to get into the patrol car, they carried out a frisk search (the hands of the respondent were placed on the hood of the car, his legs were spread and the constables patted him down); he was then taken to the police station; the respondent subsequently charged the officers with assault, alleging that they had no lawful power of arrest or search; the trial judge dismissed the charges against the officers -- he found that the arrest was lawful, as was the search, since the constables had reas gmds to conduct the search and the force used was neither excessive nor disproportionate; judge of the Superior Court dismissed the appeal on the ground that the trial judge had committed no manifest error; the Quebec Court of Appeal then substituted a verdict of guilty -- on the ground that the search had been unlawful and therefore constituted an assault, and that the power to search a person lawfully arrested is not incidental to arrest, but requires the existence of reas grounds; the Supreme Court granted leave to appeal to determine the existence and scope of the police power of search at the time of a lawful arrest-, neither of the parties raised s.8 of the Charter jW= was the search of the respondent (the frisk search) lawful? held: appeal allowed: acquittal of police officers restored --> the search was lawful at common law

MUM L'Heureux - DuM => At common law the police power of search extended to encompass a search of the surroundings of the arrest location and the seizure of anything they found there ... What must be determined, rather, is the extent to which the competing interests in the context of a lawful arrest justify a search as an incident of the arrest.. An individual who is arrested should not be able to evade the police before he is released in accordance with the rules of criminal procedure, otherwise the administration of justice will be brought into disrepute. In light of this consideration, a search of the accused for weapons or other dangerous articles is necessary as an elementary precaution to preclude the possibility of their use against the police, the nearby public or the accused himself-Further, the process of arrest must ensure that evidence found on the accused and his immediate surroundings is preserved-The legitimacy of the justice system would be but a mere illusion if the person arrested were allowed to destroy evidence in his possession at the time of the arrest.... However, the common law gave the police only such powers as were consistent with the protection of individual rights. The courts have always held that a proper balance between these two fundamental components is vital.... In this rep - ard, a 'frisk search' is a relatively non - intrusive procedure: outside clothing is patted down to determine whether there is anything on the person of the arrested individual ... The duration of the search is only a few seconds. Though the search, if conducted, is in addition to the arrest, which generally entails a considerably longer and more sustained loss of freedom and dignity, a brief search does not constitute, in view of the objectives sought, a disproportionate interference with the freedom of persons lawfully arrested. There exists no less intrusive means of attaining these objectives. A 'frisk search' incidental to a lawful arrest reconciles the public's interest in the effective and safe enforcement of the law, on the one hand, and, on the other, its interest in ensuring the freedom and dignity of individuals. The minimal intrusion involved in the search'is necessary to ensure that criminal justice is properly administered. I agree with the opinion of the Ontario Court of Appeal, as stated in R. v. Brezack, R. v. Morrison, and R. v. Miller, that the existence of reasonable and pLobable grQuads is not a Mmguisite to the existence of a Imlice W-w-&r to search. The exercise of the pgwer is not. however. unlimited. lbw VMositions can be derived: i) This power does not impose a duty. The police have some discretion in conducting the search. Where they are satisfied that the law can be effectively and safely applied without a search, the police may see fit not to conduct a search. They must be in a position to assess the circumstances of each case so as to determine whether a search meets the underlying objectives. ii) The search must be for a valid objective in pursuit of the ends of criminal justice, such as the discovery of an object that may be a threat to the safety of the police, the accused or the public, or that may facilitate escape or act as evidence against the accused. The purpose of the search must not be unrelated to the objectives of the proper administration of justice, which would be the case, for example, if the purpose of the search was to intimidate, ridicule or pressure the accused in order to obtain admissions iii) The search must not be conducted in an abusive fashion, and in particular the use of physical or psychological constraint should be proportionate to the objectives sought and the other circumstances of the situation.

A search which does not meet these objectives could be characterized as unreasonable and unjustified at common law. That is the background against which the ct must examine the facts of diis case to determine whether the frisk search conducted by the appelants constituted an assault. Needless to say, if the search in question is justified at common law, it cannot possibly constitute criminal conduct, more particularly assault. Applying the legal standard to the facts of this case, the evidence indicates that the police considered that in light of all the circumstances, it was necessary to,search the respondent. The respondents conduct, which the Court of Appeal judges characterized as 'unpleasant' and 'highly agitated and verbally abusive,' was such as to arouse a concern in the police officers for safety and effectiveness objectives. Secondly, the evidence showed to my satisfaction that the police searched the respondent in pursuit of a valid objective, here police safety in making a lawful arrest .... Third, as to the way in which the search was conducted, as already noted, it is not disputed that in the circumstances the police did not use either excessive force or constraint I accordingly consider that, as an incident to the lawful arrest of the Le ondent e f k h w jus ie , d V , th ris searc as tif an accordingly Judge Choquette made no error in dismissing the informations for assault brought against the appellants. This is sufficient to dispose of the appeal. JARREST AND DETENTION: Right to counsel

Therens [1985] S.C.R. p. 278 fg= accused lost control of his motor vehicle and it collided with a tree; police officer made a deman d for a breathalyzer test under s.235(l); accused accompanied police off to station and provided the samples of breath; at no time was the accused informed of any right to retain counsel; accused's charge of driving with excessive alcohol in his blood was dismissed bec: the trial judge held that acc had been detained and accordingly his rt under s. 10(b) had been violated (which resulted in the exclusion of the breathalyzer certificate under s.24(l)); s. 24(l) was held to include a discretion to exclude evidence where appropriate and just, independent of the duty to exclude under s.24(2) where the admission of evidence w1d bring the admin of justice into disrepute; these rulings were upheld by Sask CA; in the SCC there were five separate judgments; the ct dealt with the definition of detention and made rulings on s. 1, s.24(l),s.24(2); on the issue of detention there were two judgments -- Beetz (who simply stated that he agreed that the acc was detained within the meaning of s. 10 when the police officer administered the breathalyzer) and Ledain (whose judgment is outlined below)

1MM was the accused detained within the meaning of s. 10? bS&. appeal dismissed--> accused had been detained aki Wain ==> ... In my view, s. 10 of the Charter must necessarily refer to a great variety of detentions of varying duration, in many of which it will not be possible to make effective use of habeas corpus because the detention wil have ceased before the application can be made and determined. Habeas corpus is to he to determine the validity of a detention where that is possible. The fact that it may not be possible in some cases, bec: of the limited duration of the detention, is not, in my respectful opinion, a reason for limiting the meaning of the word "detention" to detentions of a certain duration. In determining the meaning that should be given to the word "detention" in s. 10 of the Ch it is necessary to consider the purpose of the section...The, purpose of s. 10 is to ensure that in certain situations a person is made aware of the right to counsel and is permitted to retain and instruct counsel without delay. The situations specified by s. 10 - arrest and detention - are obviously not the only ones in which a person may reasonably require the assistance of counsel, but they are situations in which the restraint of liberty might otherwise effectively prevent the access to counsel or induce a person to assume that he or she is unable to retain and instruct counsel. In its use of the word "detention" s. 10 of the Charter is directed to a restraint of liberty other than arrest in which a person may reasonably require the assistance of counsel but might be prevented or impeded from retaining and instructing counsel without delay but for the constitutional guarantee. In addition to the case of deprivation of liberty by physical constraint, there is in my opinion a detention within meaning of s.10 when a pQlice officer or other agent of the state assumes contro, over the movement of a Imrson b icant legal cg-nsQuence and which W_vgAn or imMdes access y a demand or a direction which mAy have signiE counsel.

In Chromiak this court held that detention connotes "some form of compulsory retraint." There can be no doubt that there must be some form of compulsion or coercion to constitute an interference with the liberty or freedom of action that amounts to a detention within the meaning of s. 10 of the Ch. The issue, as I see it, is whether that compulsion need be of a physical character, or whether it may also be a compulsion of a psychological or mental nature which inhibits the will as effectively as the application, or threat of application, of physical force. The issue is whether a person who is the subject of a demand or direction by a police officer or other agent of the state may reasonably regard himself or herself as free to refuse to comply... A refusal to comply with a s.235(l) demand without a reasonable excuse is, under s.235(2), a criminal offense. It is not realistic to speak of a person who is liable to arrest and prosecution for refusal to comply with a demand which a peace officer is empowered by statute to make as being free to refuse to comply. The criminal liabili1y for refusal to comply constitutes effective compulsion .... Any criminal liability for failure to comply with a demand or direction of a police officer must be sufficient to make compliance involuntary... For these reasons, I am of the opinion that the s.235(l) demand to accompany the police officer to a police station and to submit to a breathalyzer test resulted in* the detention of the respondent within the meaning of s. 10 of Ch. iMortant obiter: Although it is not strictly necessary for the purposes of this case, I would go further. In my opinion, it is not realistic, as a general rule, to regard compliance with a demand or direction by a police officer as truly voluntary, in the sense that the citizen feels that he or she has the choice to obey or not even where there is in fact a lack of statutory or common law authority for the demand or direction and therefore an absence of crim hab for failure to comply with it .... Rather than risk the application of physical force or prosecutri for wilful obstruction, the reas person is likely to eff on the side of caution, asssume lawful authority and comply with the demand. The eleme of psycholo"i al compulsion, in the form of a reas WgroVP~Qn- of suspgrision of freedom of choice. is enough to make the restraint of hI&M WygLqnJM. Detention may be effected without the application or threat of application of physical restraint if the person concerned submits or acquiesces in the deprivation of liberty and reasonabl. believes that the choice to do otherwise does not exist. Thomsen_X. &. [1988] S.C.R. P. 286 facts. no facts given in csbk i&We ,,; is someone under a raodside test demand (under now what is s.254 (2) CC) detained? b&U yes --> someone under a roadside test demand is detained (this holding by a seven panel ct clarified the uncertainty produced by LeDain's judgment in Tberens in which he spoke only for half the court); it was later held that there was no rt to counsel since the provision constituted a demonstrably justified limit on s. 10(b)

092i Ledain ==> I venture to restate what I perceive to be the essentials of those reasons, as they appear in my judgment in 77zerens, as follows: i) In its use of the word "detention" s. 10- of the Ch is directed to a restraint of liberty other than arrest in which a person may reasonably require the assistance of counsel but might be prevented or impeded from retaining and instructing counsel without delay but for the constitutional guarantee ii) In addition to the case of deprivation of liberty by physical constraint, there is detention within s. 10 of the Ch when a police officer or other agent of the state assumes control over the movement of a person by a demand or direction which may have significant legal consequence and which prevents or impedes access to counsel iii) The necessary element of compulsionor coercion to constitute a detention may arise from criminal liability for refusal to comply with a demand or direction, or from a reas belief that one does not have a choice as to whether or not to comply iv) Section 10 of the Ch. applies to a great variety of detentions of varying duration and is not confined to those of such duration as to make the effective use of habeas corpus possible

In my opinion the s.234.1 (1) demand by the police officer to the appelant to accompany him to his car and to provide a sample of breath into a roadside screening device fell within the above criteria. The demand by which the officer assumed control over the movement of the appelant was one which might have significant legal consequence bec, although the evidence provided by the roadside screening device could not be introduced against the appelant, it might provide the basis for a s.235(l) breathalyzer demand. For this reason, and given the criminal liability under s. 234.1(2) for refusal, without reas excuse, to comply with the demand, the situation was one in which a person might reasonably require the assistance of counsel. The crim liab for refusal also constituted the necessary compulsion or coercion to make the restraint of liberty a detention ... For these reasons, I am of the opinion that as a result of the s.234. 1 (1) the appelant was detained within the meaning of s. 10 of the Ch. A V. (1987.) Ont. C.A. p. 287

accused was convicted of murder; he alleged that trial judge had erred in receiving certain statements he had made to police; acc alleged that statements had been taken in violation of his rights under s. 10 (b); a police officer had telephoned the acc (a friend of the deceased) and asked if he could come over and ask him a few questions or whether the accused would prefer to come to the station; the acc decided to go to station and there he gave an exculpatory statement; a few days later he was asked to come again and he added a few facts to his earlier statement; when pressed by the officer he said he had lied in his earlier and then he gave another exculpatory statement; the acc left and was arrested two months later; he was not advised of his rights under s. 10 (b) at either interview; trial judge ruled that he was not detained within the meaning of s. 10(b) and his statements were admitted

I-MM was accused detained within the meaning of s. 10(b) and therefore denied his right to counsel? bXJ& appeal dismissed --> trial judge was correct: accused was not detained within meaning of s. 10(b)

Latim Martin LA. => I venture to suggest that in determining whether a person who subsequently is an accused was detained at the time he or she was questioned at a police station by the police, the following factors are relevant. I do not mean to imply, however, that they are an exhaustive list of the relevant factors nor that any one factor or combination of factors or their absence is necessarily determinative in a particular case. These factors are relevant i) the precise language used by the police officer in requesting the person who subsequently becomes an acc to come to the police station, and whether the acc was given a choice or expressed a preference that the interview be conducted at the police station rather than at his or her home ii) whether the acc was escorted to the police station by a police officer or came himself or herself in response to a police request iii) whether the acc left at the conclusion of the interview or whether he or she was arrested iv) the stage of the investigation, that is, whether the questioning was part of the general investigam of a crime or possible crime or whether the police had already decided that a crime had been committed and that the acc was the perpetrator involved in its commission and the questioning was conducted for the purposes of obtaining incriminating statements from the acc v) whether the police had reas and prob grounds to believe that the accused had committed the crime being investigated vi) the nature of the questions; whether they were questions of a general nature designed to obtain information or whether the accused was confronted with evidence pointing to his or her guilt vii) the subjective belief by an acc that he or she is detained, although relevant, is not decisive, bec the issue is whether he or she reasonably believed that he or she was detained. Personal circumstances relating to the accused, such as low intelligence, emotional disturbance, youth and lack of sophistication are circumstances to be considered in determining whether he had a subjective belief that he was detained.

Dealing with the first interview.... Constable Phelan testified that he asked the appelant whether he would prefer to come to the poice station or would prefer that the police officer come to see him. Ile appelant .. said that he preferred to go to the police station. At that time the police were conducting a general investigation of the circumstances of the deceased's death. Suicide had not been ruled out .. Although the appelant during the first interview was asked whether or not he could account for his time during the afternoon of Feb 14, Constable Phelan testified that every friend of the deceased or anyone that had anything to do with her that day was asked a similar question .... In my view, the evidence points to the questioning of the appellant at the first interview as being part of a general investigation, as opposed to an accusatory questioning of the appelant where the investigation into the circumstances of the deceased's death had focused on him.

The issue... [of the second interview] is more complex. ... Even if, in a broad sense, the VNIant may have been a susWct at that time, that fact alone is not determinative of the question whether he was detained. The questions put to the appelant as to whether he had gone to the deceased's home on the aft of Feb 14 were pressing and repeated several times when Inspector Collins was not satisfied with the appelant's response. 'Me questions were not, however, accusatory.... Insp Collins testif that he did not consider the appelant to be a suspect in a murder case and the fact that he did not suggest to the appelant that he had killed the deceased or endeavour to obtain an admission that the app had killed the deceased is consistent with that statement. Suicide had not been ruled out at that time... The trial judge, in my view, was entitled on all the evidence to hold that the app was not detained within s. 10 (b) of Ch when he made the statements to the police and he was correct in so holding.

R. v. IfiLk_a (1988) B.C. C.A. p. 290

LKIL accused was convicted of murder which took place near his residence; he told a police officer at the scene that he had witnessed the murder; he agreed to go to the station to make a statement; as a result of a statement made by the acc, which the officer believed showed knowledge that only the police and assailant knew, the officer turned him over to two other officers for interrogation; acc made a number of statements on which the crown later relied; the acc had not been given his rights under s. 10(b) prior to these statements iMMr,1 was acc detained within the meaning of s. 10(b) and therefore denied his rt to counsel? hd~L appeal allowed and new trial ordered --> accused was denied his rt to counsel

ratio: MacFarlane JA ==> The appelant had been taken to the police station by Staff Sgt. Tarr to make a statement as a witness. Counsel for the appelant concedes that he was not under detention while he was with the sgt. and I think that concession is properly made. In my view, the status of the appelant changed when it appeared that he might have knowledge which only the police and the assailant had. Sgt. Tarr, on learning that fact no longer treated him as a witness. If he had treated him as a witness he would have taken a statement from him and let him go. Instead, he turned him over for questioning to experienced interrogators, Sgt. Sawatsky and Constable Scott. Then began hours of relentless questioning which resulted in a finding by the trial judge that, from a certain point, that is when the tape was turned on, the statements were not voluntary. In my opinion, the appellant was detained from the moment Sawatsky and Scott took charge of him. He was no longer free to leave. He was a su~l=t an he was being detained at the pglice station for the 12=oses of having the police imstigate a susMcted offence .... I agree with counsel for the appelant that the violation of the appelant's rights under s. 10 (a) and (b) was blatant and that the admission of such evidence would bring the administration of justice into disrepute within the meaning of s. 24(2) of the Ch (illegally obtained evidence) R v. Hawkins, (1992) Nfid. C.A. p. 292 fKb; complaint of sexual assault was laid by the daughter of a friend of the accused, at whose house he had been a frequent visitor; police contacted him about two weeks later requesting an interview at his home, palce of business or the police station; acc chose to go to police station; police advised him of the nature of the complaint and of his right to remain silent; accused agreed to make a written statement which contained inculpatory remarks; he was at no time advised of his rt to counsel; acc later testified that he was at no time given the impression that the complaint was serious or that it would amount to anything was the accused detained and was statement obtained in violation of his rts under s. I O(b)? h1hL appeal allowed --> conviction set aside: the acc had been detained and the statement obtained in violation of his s. I O(b) rights

Marshall JA ==> While suspicion will be an important element in assessing the approach taken by an investigating officer, it is not in itself enough to substantiate a detention within the meaning of s. 10(b). A police officer investigating whether an offense has been committed is entitled to question anybody, whether suspected or not, from whom it is thought that useful information can be obtained, although the officer has no power to compel an answer... Section 10 (b) does not, therefore, comprehend every situation where police interview citizens, even those in respect of whom the investigator harbours underlying suspicions. It is when these suspicions become crystallized, and the investigator's approach to the encounter is changed from a questioning of the individual to an examination with the intent to charge him or her with the offence, that a detention must be deemed to have arisen engaging that person's rights under s. 10(b)... In my view, the right to be informed of entitlement to comml must arise ... where the individual is subject to the tv coercive power of the state." It is at this moment that his or her liberty and security are placed in peril, and, at that point the Charter intends and mandates that the individual be informed not only of his or her right to silence but also be affored the additional protection of being advised of the right to consult counsel. The act creating the adversarial relationship and trigerring these individual rights can be identified in the centering by the investigative officer upon the individual as the offender and the confining of the interview to attempt to discern his or her culpability and to elicit from the individual evidence to be used against him or her. This 1&rtains even though the Vcrson interviewed mU be unconscious of any omoWon in the sense of feeling he or she has no c oice but to re=nd. Weed. one might argue that s. ION was o=gia4 desigagd to meet these situations. The compulsion or cohesion mandated by Therens is present in the individuars general feeling of constraint to address the circumstances against him or her which led that person to accede to the interview in the first place. That will generally suffice to regard that, from the individual's point of view, an ambience of compulsion permeated the entire interview and it is not necessary to establish that the incrimintating statement itself was specifically made bec the person felt a lack of choice to do otherwise. to note: The Supreme Court allowed the Crown appeal in this case with a brief oral judgment indicating only that there had been no s.10(b) violation "as on the facts of this case the accused was not detained." Yet in its judgment the Supreme Court did not declare principles upon which to decide future cases. -- it would appear that the Supreme Court has rejected the idea that detention exists whenever the focus of the police investigation is on the accused. As the next case shows, one justice has even rejected the concept of pyschological detention. R v. Elshaw, [ 199 11 S.C.R. p. 293

JU=. no facts are given in the csbk iMM is the concept of psychological detention a valid concept for the purposes of s. 10 (b)?

dissent: L'Heureux - DuM (csbk contains only her dissenting opinion on this issue) > In my opinion it is neither sound constitutional interpretation nor sound constitutional policy for the s. 10(b) rights guaranteed in the Charter to be required at every instance where a citizen may feel, rightly or wrongly, psychological compulsion in the presence of a police officer. It is clear to me that the rift Ww - anteed therein should be rendered active ... at a 12oint much later in the pLQcess - after a pgriod where the ULho have an gpp=mfty to assess the situation which confronts them, to indeabb 12ossible witnesses and gs=ts, and to confirm the initial information they receive. With respect to those who hold a contrary view, it would seem to me to be misguided to require the police to issue warnings pursuant to s. 10 (b) to all the people they meet at the scene of an accident, or, as in the case currently before us, after they receive a call and must invesitgate some kind of a disturbance, on the off-chance that someone with whom they communicate may feel some End of compulsion and make a self-incriminating statement...-To adopt a position which prevents state agents from attempting to gain any sort of information whatsoever from those persons whom, as a matter of simple common sense, they would routinely be expected to question would be to confuse constitutional vigilance with paranoia. We must be careful not to leave this common sense at the doorstep when we are called on to interpret the Charter.

R. v. Bartle (1994) S.C.C. p. 297 facts: the facts and specific rulings are considered at p.301-304 of csbk -- here Lamer J. summarizes his approach distinguishing between informational and implementational duties under s. I O(b) Lamer ==> The purpose of the right to counsel guaranteed by s.10(b) of the Ch is to provide detainees with an opportunity to be informed of their rights and obligations under the law and, most importantly, to obtain advice on how to exercise those rights and fulfill those obligations. This opportunity is made available bec, when an individual is detained by state authorities, he or she is put in a position of disadvantage relative to the state. Not only has this person suffered a deprivation of liberty, but also this person may be at risk of incriminating himself or herself. Accordingly, a person who is detained within the meaning of s. 10 of the Ch is in immediate need of legal advice in order to protect his or her right against self-incrimination and to assist him or her in regaining his or her liberty. Under s. 10(b), a detainee is entitled as of right to seek such legal advice "without delay" and upon request.

S. 10 (b) of the Charter impgses, the following duties on state autprifittes who arrest or detain a 1&rson: i) to inform the detainee of his right to retain and instruct counsel without delay and of the existence and availability of legal aid and duty counsel ii) if a detainee has indicated a desire to exercise this M to provide the detainee with reasonable opportunity to exercise the right (except in urgent and dangerous circumstances); and iii) to refrain from eliciting evidence from the detainee until he or she has had that reas opportunity, again, except in cases of urgency or danger.

The first duty is an informational one which is directly in issue here. The second and third duties are more in the nature of implementation duties and are not trigerred unless and until a detainee indicates a desire to exercise his or her right to counsel. Importantly, the right to counsel under s. 10(b) is not absolute. Unless a detainee invokes the right and is reasonably diligent in exercising it, the correlative duty on the police to provide a reas opportunity and to refrain from eliciting evidence will either not arise in the first place or will be suspended ... The rights in s. 10(b) may be waived by the detainee, although the standard for waiver will be high. eQNiO in circumstances where the .y alleged waiver has been imp!Lcit. R. v. &ydgres [1990] S.C.R. p. 298 fKtL accused (a resident of Alta) was arrested in Manitoba on a charge of murder which had occured in Edmonton 6 yrs earlier; he was informed without delay of his rt to retain and instruct counsel; when he was given opportunity to call a lawyer the accused asked if they had legal aid in Manitoba bec he cld not afford a private lawyer; the officer (who was from Edmonton) said that he imagined that they had such a system; the officer then asked whether there was a reason the accused wanted to talk to a lawyer right then and the accused said "not right now no"; accused made a number of statements; he later interrupted the questioning and requested a legal aid lawyer; the legal aid lawyer contacted by the police advised the accused not to say anything more and the interrogation ended; the trial judge held that the police should have assisted the acc by determining the availability of Legal Aid at the beginning of the interrogation when the accused had essentially requested the assistance of counsel but said that was unsure as to whether he c1d afford a lawyer; trial judge found that the accused's rights under s. I 0(b) had been violated and he excluded the statements; accused was acquitted; court of appeal set aside the acquittal and ordered a new trial

JV= were the accused's rights under s. 10(b) violated? hgW; appeal allowed: acquittal at trial should be restored --> accused rights under s. 10(b) had been violated

Latim Larner =--> Once the appelant in effect requested the assistance of counsel it was incumbent on the police officer to facilitate contact with counsel by giving the appelant a reasonable opportunity to exercise his rt to counsel. On the specific facts of this case, the court is faced with the following question: when an accused expresses a concern that his inability to afford a lawyer is an impediment to the exercise of the right to counsel, is there a duty on the police officer to inform him of the existence of duty counsel and the ability to apply for Legal Aid? In my view there is. I say this because imposing this duty on the police in these circumstances is consistent with the purpose underlying the right to retain and instruct counsel. A detainee is advised of the right to retain and instruct counse without delay bec: it is M-n arrest or detention that an accused is in immediate need of legal advice ... One of the impg= reasons for retaining legal advice without dft v= being degined is linked to the Mtection of the righl a&W= self - incrimination. This is precisely the reason that there is a duty on the police to cease questioning the detainee until he has had a reasonable opportunity to retain and instruct counsel. On the facts of the case at bar, it is clear that the advice the appelant received from the Legal Aid lawyer he spoke to was to the effect that, and this was a situation which is not always the case, he should rest on his right not to make any more statements until he spoke to a lawyer in Edmonton. In retrospect, had the appelant been informed of the availability of duty counsel or legal aid at the time that he first raised a concern about affordability, the subseqeunt interrogation may never have taken place. The failure of the police to inform the appelant of the exiswrice of legal aid or duty counsel at the time that he first indicated a concern about his ability to pay a lawyer, was a restriction of the appelant's right to counsel, insofar as the appellant was left with an erroneous impression of the nature and extent of his s. 10(b) rights. As a result, I would conclude, along with the trial judge and the &ssent at the Court of Appeal that the appelant's s. 10(b) rights were violated. jmMg=t obiter: I feel compelled to make certain comments on the broader question ... namely whether it should be part of the information component of the constitutional guarantee under s. 10(b) that accused persons should be told as a matter of routine in all cases of arrest or detention of the existence and availability of duty counsel and legal aid plans. In my view, it is consistent with the purpose underlying s. 10(b) of the Ch to impose that duty on the police in all cases of detention. I find it necessary to address this issue because otherwise, among other reasons, there is an element of uncertainty facing law enforcement officials as a result of the disposition of this appeal...

There may be cases where the detainee does not explicitly ask for or about Legal Aid, but still expresses a concern about affordability of counsel. Additionally, there may be cases where a detainee says nothing about his inability to afford counsel ... Thus, police officers would be put in the difficult position of having to judge, on the spot, whether a person has expressed concerns about affordability ... In my view then, these policy concerns in respect of making police officers' duties under the Charter clear and of ensuring that all detainees are made aware of the existene of duty counsel and legal aid complement each other, and support the view that information about the existence and availability of duty counsel and legal aid plans should be part of the standard s. 10(b) caution upon arrest or detention. Criminal Procedure: Readings Summary Charging and Compelling Attendance at Trial (pp.200-244)

Questions? Christine Sakiris (282-2981)

3. CHARGING AND COMPELLING ATTENDANCE AT TRIAL

(a) Preventative Action by Anyone

(i) Use of Force to Prevent Crime See s.27

(ii) Interfering and Detaining too Prevent a Breach of the Peace See s. 30

(b) Powers of Any Individual

(i) Information Before a Justice: Summons or Arrest Warrant Any individual can attempt to initiate a criminal prosecution by laying an information before a justice who has the discretion to decide whether the case is made out to require the accused (A) to appear before the court to answer the charge. The A can be compelled to attend by issuing (a) a summons or (b) an arrest warrant.

Several Criminal Code provisions to keep in mind: re: indictable offences - s. 795 general provisions respecting informations - s.504 form of the information - s.506 and Form 2 the content and service of a summons - s.509 and Form 6 the form and execution of an arrest warrant - s.511-514 and Form 7 the backing of a warrant - s.528

The Requirements of a Valid Information = dealt with in R. v. Pilcher R. v. Pilcher (1981) Man. Prov. Ct. Facts: * -Arthur Lobson - detective - was instructed by Crime Superintendent to go to Crown Attourney - get draft information - then go to clerk of the Court's office to have it typed and sworn. The Superintendent did not discuss the case with Lobson nor give details re: the investigation. Lobson did not discuss the case with any of the investigating officers, nor was he given any police reports on the case. Lobson would normally read the reports, which would give him the reasonable and probable grounds to believe an offence was committed by the A. # s.455 [now 504] of the Code and Form 2 require the person laying the information to have reasonable and probable grounds (R&PG) that the A committed the offence. Defence argues that Lobson did not have the requisite R&PG t/f there was a latent defect in the information which required it to be quashed. isue: Did Lobson (1) have the R&PG to believe and (2)di:d believe that the A s committed the offences charged?

Held: No. The failure to comply with the mandatory requirement of s.455 [now 5041 renders the information invalid - > the information is accordingly quashed for want of jurisdiction [simply reading the information which was provided to him was not sufficient] - the solution to the issue is the meaning of "reasonable and probable grounds believes" * defn from Herniman v. Smith: ... an honest belief in the guilt of the accused based upon a full conviction, founded upon reasonable grounds, of the existence of a state of circumstances, which assuming them to be true, would reasonably lead any ordinarily prudent and cautious man, place in the position of the accuser, to. the conclusion that the person charged was probably guilty of the crime imputed. the emphasis in this definition is on the element of "belief' • def n from R. v. Sanderson: - the informant does not personal knowledge in order to have reasonable and probable grounds for believing the A did commit an offence -- RCMP records "could be relied upon by the Informant and would be sufficient on which to base an information." • in R. v. Peavoy it was held that the A "must discharge the onus of proving by a preponderance of the evidence that the informant did not have reasonable and probable grounds. - R. v. Peavoy also sets out the duties and responsibilities of a person swearing an information as follows: The police officer [must] satisfy himself that he can personally swear to the truth of the information according to its terms (i.e. personal knowledge or reasonable and probable ground) ... He need not, of course, have personal knowledge of all the facts or even most of the facts that support the allegation; indeed much of what would be available to him will, so far as he is concerned, be hearsay. He must however, be satisfied, even if it be on the basis of reliable reports made by other persons in the course of an investigation, that there is some evidence to support the charge, that that evidence in fact constitutes reasonable and probable grounds for believing that the accused committed the offence and that he believes that the accused did so.

the court concludes that s.445 [now s.504] requires that: • the informant act in a prudent and cautious manner • the informant apprise himself of the relevant circumstances surrounding the case which he reasonably and in good faith believes to be true - the informant then conclude with a genuine conviction [on the basis of those circumstances] that the person to be charged is probably guilty of the crime. R. v. Allen (1975) Ont CA An information may be presented to another justice if the first one does not act.

R. v. Jeffi~" (1976) Ont. Prov. Ct 'Facts: - each of the accused (A) were charged with unlawfully keeping liquor for sale - each A was charged under a different statute b/c of amendments to the Acts as of January 1976 and dates of the alleged offences - defence argues that the judge is not authorized to issue a summons/warrant unless he complies with s.455.3(l)(a) - being the requirement to "hear and consider, ex parte, the allegations of an informanf' - defence argues that this requirement was not met by the judge and that as a result, he does not have jurisdiction over the person of the A.

Issue: (1) What is the. jurisdiction of the judge receiving the information? (2) What is the meaning of "to hear"?

Held: (1) A justice receiving an information must perform a ministerial function and also a separate judicial one in determining whether to issue process. (2) merely reading the information is not sufficient

- the onus of showing that an information is subject to a latent defect is on the person challenging it - i.e. the A (Regina v. Peavoy) the judge must follow 2 steps which are separate and distinct: (1) deal with the jurisdictional requirement (2) issue judicial process to compel the attendance of the alleged offender.

Re Buchbinder and Venner (1985) Ont. C.A. Facts: Buchbinder swore an information before the judge against "unknown persons that can be pointed out". The judge issued a subpoena pursuant to s.455.3 [now s.507] compelling the appearance of Venner to give evidence. Venner objected. Buchbinder conceded that the information was defective b/c it failed to name Venner or sufficiently describe him so that he may be identified. The summons or warrant could therefore not be issued on the basis of that information. However, Buchbinder maintained that the justice of the peace still had the jurisdiction to conduct a hearing and compel the witness to attend.

Issue: Did the justice of the peace have the jurisdiction to conduct a hearing and compel Venner to attend court even though the information could not form the basis for a summons?

Held: No. - the judge's function is judicial not investigative - it is not the judge's responsibility to investigate alleged crimes and determine who is responsible - that is the ftinction of the police. the information could be amended by evidence brought forth before the justice - but in order to compel attendance, the justice must "at least be confident that he is dealing with a particular person who has either been named or is described wildi sufficient specificity that he could be recognized or identified."

Arrest Wi thout Warrant Power of a Citizen Arrea: s.494(l)

494.(1) Any one rnay arrest without warrant (a) a person whom he firbds committing an indictable offence; or (b) a person who, on rea-sonable grounds, he believes (i) has committed a criminal offence, and (ii) is escaping from and freshly pursued by persons who have lawful authority to arrest that person

(3) Any one other than a police officer who arrests a person without warrant shall forthwith deHver the person to a peace officer.

NB: Wbri offences are treated as indictable for the purpose of the law of arrest by virtue of s.34(l) of the Interpretation Act

(c) Special Power of Arrest of Owners, Possessors and Those They Authorize

410-\ Wi Criminal Code provides for an extended power in s.494(2): 494(2) Any one who is (a) the owner or a person in lawful possession of property, or (b) a person authorized by the owner or by a person in lawful possession of property, may arrest without warrant a person whom he finds committing a criminal offence on or in relation to that property. (3) Any one other than a peace officer who arrests a person without warrant shall forthwith deliver the person to a peace officer

(d) Powers of Peace Officers the definition of "peace officer" is in s.2 of the Criminal Code

(1) Breach of the Peace A special power of arrest is contained in s.31 of the Criminal Code 31. (1) Every peace officer who witnesses a breach of the peace and every one who lawfully assists him is justified in arresting any person whom he finds committing the breach of the peace or who, on reasonable grounds, the police officer believes is about to join in or renew the breach of the peace. (2) Every peace officer is justified in receiving into custody any person who is given into his charge as having been a party to a breach of the peace by one who has, or who on reasonable grounds the police officer believes has, witnessed the breach of the peace. qi) Arrest Without NVarr p.210

JP Arrest w/out warrant is subject to the limitation of the appearance notice situation - s.495 (1) envisages 3 distinct powers of arrest

495. (1) A peace officer may arrest without warrant (a) a person who has committed an indictable offence or who, on reasonable grounds, he believes has committed or is about to commit an indictable offence, (b) a person whom he finds committing a criminal offence, or (c) a person in respect of whom he has reasonable grounds to believe that a warrant of arrest or committal, in any form set out in Part XXVII in relation thereto, is in force within the territorial jurisdiction in which the person is found.

Q: What is the appropriate standard for a valid arrest? - Recall constitutional minimum standards for issuance of a search warrant in Hunter v. Southam in which Dickson said that the mere possibility of evidence being found is not sufficient to warrant the issuance of a search warrant. Does the same standard apply to the standard of a valid arrest?

v. Biron (1976) SCC Facts: - A was charged with creating a disturbance in public by shouting and with resiting a peace officer after an authorized raid on a bar where the A was drinking - A refused to cooperate with the police - was verbally abusive and refused to give his name - A was arrested inside the bar, then taken outside - resisted when police tried to put him in the paddy wagon - A was acquitted of the public disturbance charge b/c the Crown could not establish that any shouting had taken place. - A argues that b/c the arrest was unlawful, he could not have been resisting arrest. (i.e. without the first charge, there is no lawful arrest without warrant)

Issue: Does the charge of resisting arrest fail b/c of the A's successful acquittal of causing a disturbance?

Held: - the validity of an arrest is to be determined in relation to the circumstances apparent to the peace officer at the time the arrest was made - NOT after the trial and appeal process. - read the word "apparently" into [what is now] s.495(l)(b) - i.e. can arrest without warrant where a peace officer finds or apparently finds an individual committing an offence there is no need to read in the words "on reasonable grounds" into paragraph (b) - they are in Para (a) and (c) but were never in Para (b) - so why add them now?

QSote: in Roberge v. R. 1 (1983) SCC. Lamer interpreted Biron - 'apparently committing" -to be equivalent to having reasonable and probable grounds for believing an offence is being committed.

R. v. Storrey (1990) SCC Facts: the A argued unlawftil and arbitrary arrest

Issue: was the arrest unlawful and arbitrary?

Held: No. The officer had reasonable and probable grounds for the arrest. - Under [what is now] s.495(l) - the police are required to have R&PG that the A committed the offence (of aggressive sexual assault) before arresting him -, It is even more important to demonstrate R&PG where there is an arrest w/out warrant

9 There is a need to balance the individual right to liberty and the need for society to be protected from crime. In order to meet this balance - it is not necessary to establish more than R&PG. - It is NOT sufficient for a police officer to personally believe there are R&PG - must have objective as well as subjective belief "*** the Criminal Code requires that an arresting officer must subjectively have Qreasonable and probable grounds on which to base the arrest. Those grounds must, in addition, be justifiable from an objective point of view. That is to say, a reasonable person placed in the position of the officer must be able to conclude that there were indeed reasonable and probable grounds for the arrest." (iii) Arbitryry Detention (s.9 of the Charter) - there is a constitutional dimension to the law of arrest in sections 9 and 10(a) of the Charter - s.9 everyone has the right "not to be arbitrarily detained or imprisoned s. 10 everyone has the right "on arrest or detention" (a) to be informed promptly of the reasons therefor"

R. v. Duguay (1985) Ont. C.A. [contrast w/ Iron re: the meaning of "arbitrary detention"] Facts: * "experienced" police officers arrested 3 A on suspicion of involvement in a residential burglary - victim said they were in neighbour's yard the night of the burglary asking whether they usually put their dog in the garage - Neighbour identified one of the A - police asked him and his friends from the night before to come over - where the victims recognized 2 of the 3 A.- arrest followed -As were read their rights in the cruiser, subsequently gave inculpatory statements - police agreed that at that point there was no evidence - the purpose of arrest was to further the investigation by obtaining inculpatory statements and fingerprints the arrest was unlawful (not contested)

Ussue: Is an unlawful arrest necessarily arbitrary under s.9?

Held: NOT every unlawful arrest = "arbitrary detention", but the absence of R&PG could support an inference of arbitrary detention. • Test: consider (1) the facts of the case (2) the extent of the departure from the standard of R&PG and the honesty of the belief and basis for the belief in the existence of R&PG on the part of the person making the arrest. • the arrest in this case WAS arbitrary b/c it was for an improper purpose - Dedman forbids arrest for the purposes of investigation - there was neither R&PG nor an honest belief in R&PG

Dissent: * to believe on R&PG that a person has committed an indictable offence does not require that the grounds be made up of evidence that can be later adduced in a court room - a detention based on something less than R&PG is not necessarily arbitrary - in this case the detention "was neither capricious nor random"

Iron v. R. (1987) Sask C.A. Facts: - police officer on routine patrol stopped A on rural highway to check documents C) - s.83(7) of the Vehicles Act of Saskatchewan says that driver must stop when signalled - A didn't have a valid license - charged and convicted accordingly

Issue: Was this "arbitrary detention"? What is the meaning of 'arbitrary detention"?

Held: - the stop was arbitrary - an illegal detention is necessarily contrary to s.9 -- the justification of the detention is to be left for a s. 1 analysis - the essence of arbitrariness is capriciousness in an lack of a reasoned foundation for interfering with the right to be left alone (this right is at the center of s.8 and 9 of the Charter)

R. v. Simpson (1993) Ont. C.A. Facts: * Internal police memo re: suspected crack house - police saw woman leave car parked in the driveway - go into the house - then come out with a man who became the passenger in her car - Police officer - without any information on these two people other than his observations - followed and stopped them - seeing bulge in A's front pocket - asked A to take it out - it was cocaine C) Issue: Were the A's s.8 and s.9 Charter rights infringed?

If-"Xeld: There was a violation of s. 9 -- b/c the detention was not authorized by law, s. 1 does not apply lawfulness is-not determinative of arbitrariness BUT "If the detention is lawful, it is not arbitrary unless the law authorizing the detention is arbitrary. If the detention is found to be unlawful, that finding will play a central role in determining whether the detention is also arbitrary." (p.224) - where the p.o. conduct interferes with the liberty or freedom of the individual -that conduct must be authorized - the p.o. conduct in this case was not authorized by statute - t/f must look to common law At common law: - Duguay: does not stand for the principle that any and all detentions for investigative purpose constitute a violation of s.9 Dedman: in exercising general duties - police powers must be authorized by law - there is no right to detain a person for questioning or further investigation (no physical restraint except as authorized by law. Dedman does not mean that you cannot detain in investigation unless the police have the power to arrest - it means that the desire to question or otherwise investigate an individual is not in and of itself authorization for detention of that individual so - Dedman stands for the proposition that there is no general power to detain whenever that detention will assist a p.o. in the execution of his or her duty (~his does not mean that deny detention for investigative purposes in all circumstances short of arrest) Therens; Thomsen: police to detain individuals for investigative purposes when they have no basis to arrest them

the power to detain for investigative purposes is regulated by the common law and criteria in Waterfield: (1) Were the police acting in the course of their duty? note: the police's "stated purpose" is not determinative of the question (2) was there an 'articulable cause" for the detention? - "cases require a constellation of objectively discernible facts which give the detaining officer reasonable cause to suspect that the detainee is criminally implicated in the activity under investigation" - the facts must meet the objectively discernible standard (StorTey) - the presence of articulable cause alone does not render any determination a justifiable exercise of common law powers --- it is only the first steJ2 in the inquiry set out in Dedman and Waterfield

the attendance of these individuals may go to establishing an articulable cause but in this case the constable did not have an articulable cause since there was no articulable cause, there was no common law power to authorize the detention - the detention was unlawful and arbitrary Duguay: if detention is unlawful, it is not arbitrary IF the officer erroneously believed on reasonable grounds that he had an articulable cause - this does not apply in this case - U b/c the detention was not authorized by law, s. I of the Charter does not apply

R. v. Hufsky (1988) SCC & R. v. Ladouceur (1990) SCC - these cases held that a "detention in the unfettered discretion of the police officer is necessarily 'arbitrary' contrary to s. 9. it must be demonstrably justified as a reasonable limit under s. I - cases dealt with the constitutionality of spot checks under s.216 of the Ontario Highway Traffic Act s.216 (1) A police officer, in the lawful execution of his duties and responsibilities, may require the driver of a motor vehicle to stop and the drier of a motor vehicle, when signalled or requested to stop by a police officer who is readily identifiable as such, shall immediately come to a safe stop. (2) Every person who contravenes subsection (1) is guilty of an offence and on conviction is liable to imprisonment for a term of not more than six months, or to both.

Hufsky: even though the detention is authorized by law, it is arbitrary b/c there is no criteria for selection of the drivers to be stopped and subjected to spot check procedures -this discretion is arbitrary when there is no criteria expressed, either explicitly or implicitly. C) (iv) Reasons For Arres - the common law requirement re: the duty to provide a reason for arrest = set out by the House of Lords in Christie v. Leachinsky (1947) - the SCC pronounced on the issue in Gamracy where they held the police duty to inform the A of the reasons for his arrest is exhaustively codified in s.29(2)of the Criminal Code: 29. (2) It is the duty of everyone who arrests a person, whether with or without warrant, to give notice to that person, where it is feasible to do so, of (a) the process of warrant under which he makes the arrest, or (b) the reason for the arrest

Gamracy v. R. (1974) SCC Facts: - peace officer tried to arrest the A for outstanding warrant (did not know the offence under which that warrant was issued) - he called out to the A who was about to enter into his house, but A ignored him - peace officer grabbed the A's arm as = opening the door and told him he was under arrest for outstanding warrant * altercation ensued - p.o. phoned for reinforcements but did not ask the nature of the charge in the outstanding warrant. more violence resulted - officer injured Issue: was it a valid arrest?

(~eld: it was a valid arrest w/out warrant • officer = acting under the authority of s.450(l) [now 495(l)] • b/c -the officer did not have a warrant in his hand s.29(2) applies - read para (a) and (b) disjunctively - the duty was discharged by telling the A that the reason for his arrest was the outstanding warrant - it is not feasible nor part of the officer's duty to obtain the warrant to show A or ascertain its contents - A can't complain that he did not get the information required when he produces the situation which makes it practically impossible to inform him - i.e.through counterattack or by running away (Viscount Simon)

Dissent: - agrees that 29(2)(a) and (b) are alternative and disjunctive requirements BUT the duty is not discharged under either paragraph "by simply informing the person whom he is arresting that there is some kind of warrant 'out for him'." (p.229) in this case it was feasible for the p.o. to have told the A of the specific charge in the outstanding warrant - the po could have checked it either upon first learning of the outstanding warrant or when phoning for reinforcements - b/c the p.o. did not meet the duty under s.29(2) he was not acting in the course of his duty when he attempted to arrest the A - t/f the A cannot be convicted of assaulting a p.o. in the execution of his duties.

Wote: what is the impact of s. 10(a) of the Charter have on the majority decision?

(v) Ent[y Into Premises: Q: Do the police have the power to enter premises to arrest? A: the law is vague Ouimet Report stated that the police can enter premises, including a dwelling house, by force if necessary, and without a warrant in order to: (1) prevent the commission, of an offence which would cause immediate and serious injury to any person, if he believes on reasonable and probable grounds that any such offence is about to be committed (2) effect the arrest of a person who has been found committing a serious crime; and who is being freshly pursued and who seeks refuge in such premises.

R. v. Macooh (1993) SCC Facts: * A ran stop sign - ran more when p.o. pursued - stopped and ran towards back door of apartment - ignored police call to stop and entered the apt. - p.o. went to back door of apt. and called for him - he was ignored - police identified himself as RCMP officer and entered - told A = under arrest for failure to stop for p.o. - A refused to get dressed - altercation ensued in which p.o. observed signs of inebriation - p.o. demand for breath sample refused - A = charged w/ impaired driving, failing to stop for a p.o., failing to submit for a breathalyser test and assaulting a p.o. w/ intent to resist arTest.

Issue: - did the police officer have the right to enter into a dwelling-house to arrest to A (the p.o.'s authority to arrest w/out warrant is not in issue) - narrow issue: could the "hot pursuit exception" be extended to cases for arrest w/out warrant for provincial offences?

Held: police officer was authorized to enter the house - the "hot pursuit exception" is extended to cases for arrest w/out warrant for provincial offences (the court did not decide on the existence of the power in cases where there is no hot pursuit for a provincial offence) court looks at how the exception developed: - at common law: Eccles v. Bourque (1975) SCC - the sanctity of the home is subject to certain exceptions - including the right to make arrest (yield to public interest) provided (1) there are R&PG for the belief that the person sought is w/in the premises; and (2) the officer has made proper announcement before entering - extension to indictable offences: R. v. Landry (1986) SCC - the court extends the exception to arrest w/out warTant for offences under [what is now] s. 495(l) i.e. indictable offences provided that the standards set out in Eccles v. Bourque are met: ASK: (1) Is the offence in question indictable? (2) Has the person who is the subject of the arrest committed the offence in question or does the peace officer, on R&PG, believe he or she has committed or is about to commit the offence in question? (3) Are there R&PG for the belief that the person sought is w/in the premises? (4) Was proper announcement mad before entry? Need an affirmative answer to ALL these questions in order for the arrest to be lawful Court look to see if the exception can be extended further: - def n of hot pursuit = "continuous pursuit conducted w/ reasonable diligence, so that pursuit and capture along with the commission of the offence may be considered as forming part of a single transaction" - in this case, there was hot pursuit ihe common law recognizes a right of entry for hot pursuit for indictable and other types of offences (provided there is real continuity between the commission and pursuit - i.e. an officer can take over a pursuit started by another officer) - there is no reason to alter that common law rule - the distinction between indictable and other categories of offences does not reflect the severity of the offences in question and is t/f an excessive limit on police powers - the court rejects the distinction in this context. the power of entry is limited to cases where the officer is authorized to arrest W/out warrant - the officer was in this case t/f the p.o. was authorized to enter and make the arrest.

(vi) Meaning'of Arrest: R. v. Whitfleld (1970) SCC Facts: * A has outstanding warrant out for him - police officer goes up to car at red light -says to driver - I have a warrant for your arrest - stop the car and shut off the ignition - the light turns green and the driver starts driving - at next light the p.o. catches up with the car - reaches in tying to grab the ignition key - grabbed the shirt of the driver and said "you are under arrest - stop the car" - A sped up and kept going -officer thrown from the car.

Issue: Could the A be charged with escaping lawful custody? i.e. was there a lawful arrest?

Held: there was a lawful arrest * court rejects the argument that touching the A is not sufficient to constitute an arrest and that the Crown must establish the beyond a RD that constable had captured the A and then broke away - an arrest is defined as follows: "Arrest consists of the actual seizure or touching of a person's body with a view to his detention. The mere pronouncing of words of arrest is not an arrest unless the person sought to be arrested submits to the process and goes with the arresting officer. an arrest may be made either with or without a warrant."

(vii) Appearance Notice: • the Bail Reform Act was aimed at minimizing arrest and detention prior to trial • s.495(2) of the Code was part of this effort - designed to limit the power of arrest 495. (2) a peace officer shall not arrest a person without warrant for (a) an indictable offence mentioned in section 553, (b) an offence for which the person may be prosecuted by indictment or for which he is punishable on summary conviction, or (c) an offence punishable on summary conviction, in any case where (d) he believes on reasonable grounds that the public interest, having regard to all the circumstances including the need to (i) establish the identity of the person, (ii) secure or preserve evidence of or relating to the offence, or (iii) prevent the continuation or repetition of the offence or the commission of another offence, may be satisfied without so arresting the person, and (e) he has no reasonable grounds to believe that, if he does not so arrest the person, the person will fail to attend in court in order to be dealt with according to law. s.495(2) is very complex - interpretation by the courts has been difficult, police are in an awkward position to make a decision on the spot.

The appearance notice must be in the proper form (s.501) supported by an information laid before a justice as soon as practicable (s.505(a) and s.508). The justice must cancel or confirm the appearance notice. Technical errors in an appearan * ce notice, or a delay in its confirmation by an information are no longer sufficient to render the notice a nullity. Technical errors in the appearance notice can be corrected as follows: errors may be cured by a valid information [R. v. Littlejohn (1981) B.C.C.A.; Re Thompson and R. (1984) Alta CA] - lack of a timely supporting information can be cured by the A attorning to the jurisdiction by his appearance [R.. v. Naylor (1978) Ont. C.A.1 * even after an appearance notice has lapsed, a valid information can still be sworn and the accused's appearance compelled by way of summons [R. v. Tremblay (1982) B.C.C.A.]

s.495(3) enacts a wide justification - s.495(3)(a) has been interpreted to conclusively deem an arrest contrary to s.495(2) as legal for all purposes connected with criminal proceedings [R. v. Adam; R. v. McKibbon; R. v. Tully] 495. (3) Notwithstanding subsection (2), a peace officer acting under subsection (1) is deemed to be acting lawfully and in the execution of his duty for the purposes of (a) any proceedings under this or any other Act of Parliament, and (b) any other proceedings, unless in any such proceedings it is alleged and established by the person making the allegation that the peace officer did not comply with the requirements of subsection (2). note alternative interpretation to s.495(3) on p.244 Right to Counsel

R. v. Bmtle [p. 3011 (1994), 33 CJL (4th) 1 (S.C.C.)

Facts: - the accused failed a breathalyser test taken in early hours of morning -was arrested for impaired driving. -was read his rights under s. I O(b) of the Charter - was informed of the availability of legal aid, but not of the free and immediate legal advice available from duty counsel which could be reached by a toll-free number. - the number was on the police caution card. - both at the roadside and at the station the accused was asked if he wanted to call a lawyer; both times declined.

Issue: Was the defendant's right to counsel under s. 10(b) of the Charter violated?

Held: Appeal allowed. The accused's right to counsel was infringed: he was not properly informed of the existence and availability of duty system and how to access it.

Ratio: Larner C.J. information on the right to counsel provided to a detainee must be comprehensive in scope and delivered in a timely and comprehensible manner. -detainee must be advised of available legal services before he can be expected to assert his right to avail himself of them. -s. 10(b) entitles detainee to be informed of any free, preliminary legal advice system and how this legal advice can be accessed. -this may have misled the accused about the nature and extent of his right to counsel. -there was no valid waiver by the accused of his right to counselWaivers must be clear, unequivocal and with the full knowledge of the rights the procedure is designed to protect.

-caution given to him on both occassions failed to convey the necessary sense of immediacy and universal availability of legal assistance. -even if there were no phones available at the site of the arrest the accused should have been told the toll free number.

R. v. MwWnen [p. 3051 (1987), 58 C.R. (3d) 97 (S.C.C.)

Facts: - the accused was arrested for the robbery of Mac's Milk Store, as well as the theft and possession of a car used in the robbery. -based on information received, the plain clothed police officers awaited the accused at E &R Simonizing. Here, one of the arresting officers found a gun protruding from under the seat of the car. The accused's car matched the description of the car used in the robbery. The robber had used both a gun and a knife in the robbery. He has wom a grey hooded sweat shirt. the police officers arrested the accused, searched and handcuffed him,

Af~ the accused was read his rights from a printed card the police carried. When the accused responded "It sounds like an American T.V. program" the police reread the card. - the accused asserted his right to counsel. - the officer began to question him regardless. ("Where is the knife that you had along with this (the gun)when you ripped off the Mac's Milk ... Y - the accused made incriminating statements ("He's lying. When I was in the store I only had the gun. The knife was in the tool box in the car.") - these statements relied on by trial judge in his conviction. - the knife and sweatshirt were uncovered from the car as well. - again, the accused asserted his right to speak to a lawyer. - there was a phone in the office which the police made use of throughout the afternoon, but which was never offered to the accused to call his lawyer, nor did he request it.

Issues: Although the accused was properly informed, were there correlative obligations on the police once the accused has asserted his right to counsel?

Was the incriminating evidence improperly obtained in violation of the accused's charter rights (10(b))?

Should this evidence be excluded?

Held: In addition to the duty of the police to be inform the accused of the right to counsel, s. 10(b) requires the police to provide a reasonable opportunity for the accused to exercise this right and retain counsel without delay, as well as to cease questioning or attempt to elicit evidence until the accused has been given this reasonable opportunity.

The evidence was improperly obtained and hence, should be excluded.

Ratio: Lamer J.

- The accused's right to be given a reasonable opportunity to exercise his right to retain counsel without delay was clearly infringed here. The accused did not have to request to use the phone in order to have this right. There was no urgency in this situation to justify the accused not being able to contact counsel.

- His right to not be questioned until he has been given a reasonable opportunity to retain counsel was similarly infringed. Immediately following his assertion to exercise his right to counsel, the police officers commenced questioning in a form designed to illicit involuntary answers.

- This was not a case where urgency required the police to proceed without the accused having a reasonable opportunity to retain counsel.

- Further, the accused did not implicitly waive his right. The standard of implied waiver is very high. Baig v. R 1p. 3091 (1987), 61 C.R. (3d) 97 (S.CC)

Facts:

- the accused was charged with murder. - entered police vehicle. Was informed of his rights including the right to counsel. - there was no conversation en route to the station. - at the station, the accused was questioned. ("Anees and Raza have been arrested and we know exactly what happened so you may as well tell us about it.") But before he answered he was reminded of his rights. - the accused then made incriminating statements. Typed on statement form. Accused read, signed statement which included questions as to whether the accused understood the charge, the caution, and his right to counsel. - Was acquitted at trial by a directed verdict when the judge excluded evidence under s. 24(2) of the Charter due to a violation of the accused's right to counsel under s. 10(b). - The Court of Appeal reversed the finding.

Issue: Must the accused assert a right to counsel before correlative obligations (for eg: to desist from questioning the accused) are placed on the police?

Held: Yes. (Court of Appeal decision affirmed.)

Ratio:

- the accused was not denied the opportunity to ask for counsel. - the accused did not assert this right. - once the police advises the accused without delay of his right to counsel under 10(b) no other duties are triggered unless the accused asserts he wants to avail himself of this right.

Lechdr Y. R I P. 3111 [1989146 C.CC (3d) 129 (S.CC)

Facts:

-three youths arrested in the middle of the night. - charged with break, enter, theft. - were properly advised of their right to counsel. - at 2:00 a.m. they tried unsuccessfully to reach their lawyer. - refused option to try to make contact with another lawyer. - approx. I hr. later the boys partook in a line up. - they were not informed that they were not obliged to participate.

Issues: Did the police fulfil the obligation to give the detainees a reasonable opportunity to exercise their right to counsel?

Did the police also fulfil their obligation to desist from questioning and gathering evidence until the opportunity was given? Held: The police failed in both these duties. Appeal allowed.

Ratio: Lamer J.

-The fact that the appellant Leclair said that he did not wish to call another lawyer does not constitute a valid waiver of the right to counsel. ( since he asserted his right onus on Crown to show it was clear and unequivocal waiver.) - The appellant Ross was not even given this option. - They asserted the right to retain counsel of their choice. Although this right will not subsist in all situations (such as where the lawyer won't be available for an unreasonable period of time), it was fair to assert it here. The accused must always be diligent in trying to obtain counsel of his or her choice, otherwise the correlative obligations on the police (to desist from questioning and gathering ev.) suspended. (In order to prevent abuses of this right which could result in impeding investigations indefinitely, as well as interference with the administration of justice.) They were not afforded a reasonable opportunity to exercise right to counsel. Police just had to wait a few more hours until normal business hours. - Police should have refrained from taking further steps and didn't. Appellant told to line up without reasonable op to retain counsel and get advice as to obligation to partake in the line up. If he had, would have been told that although there was no legal or statutory obligation on him to participate, not doing so could prejudice him. Further, the fact that he participated does not constitute a waiver either.

R. v. Burfinghmn [page 3151 (1995), 38 C.R. (4th) 265 (SX-C.)

Facts:

- accused arrested on charge of murder of B.H. - confessed to this murder. - police suspected him of murder of D.H. since it was committed in similar manner. - over 3 day period, acc. subject to intensive, often manipulative interrogation. - throughout he repeatedly stated he would not speak until he spoke to his lawyer. - 4th day: offered a "one time"deal available only until end of weekend (during which his lawyer was away): would only be charged with second degree murder if he disclosed the location of the gun as well as other info. - he did get advice from another lawyer (not to talk to the police) but accepted the deal. - fulfilled his end of the bargain. The police reneged: he would be charged with first degree murder but could plead not guilty. - at trial the confession and location of gun excluded. But gun itself and incriminating statements to girlfriend after deal included. - convicted of Ist degree murder of both. - B.C.C.A. dismissed the appeals.

Issue: Was the appellant's ni ght to counsel under I O(b) of the Charter violated?

Held: right to counsel was violated. Appeal allowed. Ratio: Iacobucci J.

- appellant's rights were violated in several wa ys: 1) police continued to question him despite his assertion of the right to counsel. 2) 10(b) also prohibits police from belittling accused's lawyer with intention of undermining confidence in the relationship. 3) 10(b) also infringed by the improper conduct by the police regarding the plea bargaining: the offer should only be made to the accused's lawyer or to the accused in presence of lawyer, or deal should have been kept open until the ap. could have talked to lawyer. - fact that he did talk to a lawyer insufficient to discharge police of duties in light of the seriousness of the situation (plea bargain to be considered.) - badgering resulted in misunderstanding as to right to counsel. Must take positive steps to facilitate understanding. - ap. wouldn't have agreed to deal if not for concerted efforts of police to dissuade him from consulting counsel. - no urgency in this situation to justify acting as they did, curtailing right to counsel.

[Note p. 318: R v. Prosper]

No constitutional obligation on governments to provide free duty counsel system upon arrest or detention. But in absence of such a system, the police may have to "hold off' from questioning... longer in order to allow detainee to get counsel.

R v. Chv*son 1p. 3181 [1986125 C.C.C. (3d) 207 (SX-C.)

Facts:

-accused very intoxicated when arrested, charged with husband's murder. - was given a police warning, informed of right to counsel to which she responded there was "no point" in having counsel. - underwent questioning while in this state (drunk, emotional) despite aunt trying to stop it until counsel could be retained. - at trial: statement excluded since she didn't appreciate the consequences of statement. - C.A. reversed this decision.

Issue: Was the accused's right to counsel infringed? If so, should the confession be excluded under 24(2) of the Charter?

Held: Yes. Appeal allowed.

Ratio: Wilson J.

- alleged waiver of rt. to counsel must be carefully considered since purpose of right is to ensure the accused is treated fairly in the criminal process. - accused's awareness of consequences of what he/she is saying Is crucial. - validity of waiver dependant on it being clear, unequivocal that person is waiving procedural safeguard with the full knowledge of rts the procedure in designed to protect and the effect the waiver will have on these rights (Korponay) - waiver of 10(b) rts by intoxicated person must pass some form of "awareness of the consequences" test. - here, the test was not passed. - police should have waited until she was in sufficiently sober state to properly exercise her right to counsel or be fully aware of the consequences of a waiver. - was an improperly obtained confession. Was a blatant violation of rts. Questioning while she was still drunk aimed at extracting confession which they might not have been able to get later. - should be excluded under s. 24(2) otherwise police would be invited to disregard charter rights of citizens and do so with impunity. Also admin. of justice would be brought into disrepute.

P. v. Smith [p. 3211 [1989] 50 C.C.C- (3d) 308 (S.C.C.)

Facts:

- accused arrested at 7:00 p.m. at home on charge of robbery which had taken place 5 mths previous. - was properly informed of right to counsel. - for approx. 2 hours they made various stops at acc's request. - at station, requested opportunity to contact lawyer. Police provided him with a phone and phone book. - decided not to call since offices closed. - approx. I hr. later began questioning. - acc again asserted rt. to counsel but questioning continued. twice more asserted that he would not speak until he had spoken to lawyer. - eventually made a statement "off the record" which was later admitted at trial. - B.C.C.A. upheld trial judge's decision that there had been no violation of s. 10(b).

1ssue: Was the statement obtained in violation of the accused's right to counsel and should it have been excluded?

Held: No, the statement was not obtained in violation of s. 10(b) and hence there was no error in the lower courts in allowing the statement as evidence.

Ratio: Lamer J., (Gonthier J. concurring)

the correlative obligations imposed on the police once the accused asserted his right to counsel were suspended when he failed to exercise this right diligently. - such limits are necessary since investigations could otherwise be delayed indefinitely. - fact that the accused failed to even try to call his lawyer was fatal. He was not reasonably diligent.

(per L'Heureux-Dub6 J., concurring) -s. 10(b) was not infringed since in view of the circumstances of time and place and the reasonable behaviour of the police, acc. had reasonable opportunity to communicate with counsel of his choice, but he failed to avail himself of it. (per Sopinka J., concurring) - this case was close to the line. - acc was casual in asserting his right. He "frittered" away 2 hrs, worrying about comparatively trivial matters during which time counsel would have been more easily contacted.

(LaForest I dissenting (Dickson CIC., Wilson I concurring.))

- the accused did not waive his right to a lawyer-, he positively and repeatedly asserted desire to exercise his right. - the police made efforts in questioning the acc, not to involve the lawyer as well as appealed to the acc.'s sense of guilt (by mentioning his mother, his son) - fact that he answered questions does not constitute a waiver. - no awareness of consequence of speaking in absence of lawyer. - no reasonable opportunity to contact counsel: he did not make phone call reasonably thinking that at that time of night there would be no one in offices to receive the call. - there was no urgency in this case to question the accused. esp. since crime happened 5 mths; earlier. - doesn't accept distinction between this case and Ross (Leclair) (that in this case the accused had at least tried to call.) there was here a wilful violation of appellant's rights.

d-11, k.)

11-~ ------Discovery

Royal Commission on the Donald Marshall Jr. Prosecution [p. 5551

- Ours is a "criminal justice system" not a "criminal convictions system."

- As a representative of the state, the prosecutor is responsible for seeing that the State's system of law enforcement works fairly. Nowhere is this more apparent than in the Crown's obligation to disclose to the accused.

Ihe failure of the Crown to disclose to Donald Marshall was a critical factor in his case and contributed to his wrongful conviction and continuing imprisonment.

Anything less than complete disclosure by the Crown falls short of decency and fair play.

The extent of disclosure is a vexing question, requiring the balancing of the interests of the state in the administration of justice, versus the right of the accused to a fair trial.

legitimate concern of the prosecutor: that the information disclosed can result in justice not being done.

- But access to info necessary for defence to prepare case.

- Pre-trial disclosure duty ought to be carried out in manner consonant with rather than inimical to the prosecutor's role as a representative of the Minister of Justice.

Thus, info. must be given in a timely, fair manner, to appraise the accused of the case which must be met, and enabling the accused to make a full answer and defence which is one of the principles of fundamental justice in s. 7 of the Charter.

-Ad hoc case by case approach to decide whether failure to disclose infringed accused's rights not adequate. As much discretion and subjectivity as possible should be removed from the disclosure duty. Statutory ob on Crown to disclose is required

-All information known to the Crown might be considered useful to the accused. Whether or not defence asked for the info should be irrelevant. The Crown should not hold back evidence because it will assist the accused. Duty should be for pros. to bring forward every material fact known to them, whether favourable to the accused or not,

- All parties must participate on equal basis for system to work. This means the defence must have access to the same info. the Crown does.

(Note: this report was one of the sources relied on by the S.C.C. in recognition of a constitutional right to discovery under s. 7.) R v. Sfinchcombe [p. 5601 [1"113 S.CJL 326 (S.CC.)

Facts:

- in the preliminary inquiry, a witness gave evidence favourable to the accused. - later the prosecution interviewed the witness, but decided not to call the witness. - defence applied for the disclosure of these statements. - the trial judge refused the motion. - the C.A. upheld this decision.

Issue: Was the prosecution obliged to disclose these statements?

Held: Yes.

Ratio: ( Sopinka J.)

- civil proceedings: element of surprise has been sacrificed for principle that justice is better served when the element of surprise is eliminated and the parties are prepared to address the issues on complete information of the case to be met. Unfortunately it has lingered on in criminal proceedings. - (at time of this case) the pros. does disclose on voluntary basis but no comprehensive rules making it mandatory exist. A -fruits of the Crown's investigation not to be used to convict, but is property of the public to be used to ensure justice is done. - the defence has no duty to assist the pros. - pros should have some discretion as whether to disclose (though this should be subject to review) clearly irrelevant evidence (but must err on side of inclusion); identity of certain witnesses to protect them from injury, harassment; informers (privileged); timing of disclosure to complete investigation. - info ought not to be withheld if will impair rt. of acc. to make full answer and defence (one of the pillars of the crim. justice system.) - duty of defence: to bring to the attention of court at earliest possible time failure of pros. to disclose. - duty of disclosure applies to all offences. - timing of disclosure: before acc. called on to elect mode of trial, plead, since needs to know strengths and weaknesses of case before committing on these issues. - request should be complied with so as to give sufficient time for def. to consider info before pleading, electing. - this is a continuing ob (must turn over any new info.) -all relevant info should be disclosed subject to the reviewable discretion of the Crown and not only that which the Crown intends to introduce into evidence. There is to be no distinction btwn. inculpatory and exculpatory evidence. - all statements obtained from persons who have provided relevant info. to authorities should be produced notwithstanding fact that they won't be called as Crown witnesses. If no statement, then notes should be provided, if no notes then names, address, occupation of witness. - tactical advantage must be sacrificed in interest of fairness and ascertainment of facts of the case. R v. OiConnor [p. 5691 (1996), 44 C.R. (4di) I (S.CC)

Facts:

Catholic bishop charged with 4 counts of sexual assault on aboriginal women approx. 25 yrs previous. (note: he was convicted about 1 mth ago) - the defence wanted the disclosure of records of the complainants which were in the hands of third parties (medical records, school records, counsellors) - Crown said that they hadn't disclosed these by inadvertence (though question of bad faith on their part was raised) and later, when realized, because they didn't want to harm the complainants. Crown also argued that there was a gender bias. - the trial judge granted a judicial stay (tantamount to an acquittal) because the prosecution hadn't properly executed disclosure duty.

Issue: Does the disclosure duty extend to confidential records in the hands of third parties?

Held: Yes, although a two step test must be met. (Here, the defence failed to meet the test, but can try again.)

Ratio:

Lamer C.J. (Sopinka, Cory, lacobucci, Major JJ. concurring)

- must balance Charter values and rights. The right of the accused to make full answer and defence vs. right to privacy. - Ist step: (threshold) onus on accused to satisfy judge the information in these records is likely to be relevant. (goal to prevent fishing expeditions) - 2nd step: the production procedure= (higher test of relevance)accused must bring formal written application supported by affidavit setting out specific grounds for production - notice must also be given to the third parties and person who has privacy interest at stake - upon production, judge must examine the records, determine whether, to what extent they should be released to accused. Must decide whether non production order would be a reasonable limit on accused's night to make full answer and defence. -since Crown can always obtain records of third parties by search warrant, fairness requires accused be treated on equal footing. - when records in the hands of the Crown, relevance presumed and Charter interests need not be balanced. Also, complainant taken to have waived rights.

L'Heureux-Dub6 J. (LaForest, Gonthier, McLaghlin JJ. concurring)

- sets out two step test as well. - I)relevance of info: if accused cannot demonstrate likely relevance of information, the application should be refused. - 2) in deciding whether to produce the records, discretion must be exercised in a manner consistent with Charter values of right to privacy and equality without discrimination. - factors to be considered: extent to which record is necessary for accused to make full answer and defence; the probative value of the record; the extent of reasonable expectation of privacy of that record; whether production premised on discriminatory belief or bias; the potential prejudice to the dignity of the persoz occasioned by the production of the record. (Lamer CIC. et al agreed with these considerations.) - other factors: extent to which release would fi-ustrate social goal to have sexual offences reported and for victims to seek treatment. (Lamer: other avenues open to judge to ensure this) Effect on the integrity of trial process by producing or failing to produce the record. (Lamer: this should be dealt with at admissibility and not production stage)

fNote p. 572 LLA. v. A.B.]

-handed down same day as O'Connor. -accused charged with indecent assault of L.L.A. -subpoenas served on third party asking them to bring all records relating to proceeding and to L.L.A. Served notice of motion on third party institution and the Crown to produce all records on L.L.A. - Crown made motion to quash subpoenas. Trial judge dismissed it. - Held: the accused failed to meet the procedural requirements and substantive law as set out in O'Connor. Appeal to quash subpoenas granted. Accused may start a O'Connor law. yew following 0 Clare Mauro (842-6362) A. REMEDIES (pp.450-467)

JUDICIAL STAY AS ABUSE OF PROCESS

Prior to Charter, courts developed common law power to stay criminal proceedings as an abuse of process. Power used sparingly to control prosecutorial practices at trial considered by judges to be 'oppmssive.'

R.ourke v. R. (1978) S.C.C. case re: abuse of process at common law, pre - Chaile abuse arising from pre-trial delay by the police

FACTS: Accused charged with kidnapping & robbery alleged to have been committed in Oct.'7 1. Information sworn in Feb.73- and warrant issued. Accused arrested in Apr.'73 and committed to trial in June. In Nov.'73, accused appeared for trial before County Court judge on indictment preferred by A.G.-B.C. Accused (appellant) moved to stay proceedings on grounds that a delay in bringing him to trial created prejudice toward his defence. Acc. argued that: a) key witness died in interval between crime and trial; b) acc. had been accessible to the police and they could have arrested him sooner if they had acted with teasonable diligence. Therefore, C.C. judge stayed proceedings. Supreme Court of B.C. dismissed application for mandamus on basis that C.C. judge had acted within his jurisdiction and had properly appl'd the law to the evidence before him. B.C.C.A. unanimously reversed this decision.

ISSUE: Does a trial judge have the power at common law to stay proceedings for abuse of process? Did trial Judge err in granting a stay of proceedings in this situation?

HELD: Absence of any provision in Criminal Code or at common law to grant a stay of proceedings on grounds that arrest/trial took too long, means that this remedy not available. Pigeon, J.:, - no rule at common law that prosecution must be instituted promptly and ought not to be permitted to go ahead if delay in instituting it may have caused prejudice to accused - general rule that criminal offences are not subject to prescription except where specifically est. by statute - them is no general discretionary power to stay proceedings on grounds that prosecution is oppressive - principle expressed in D.P.R v. Humphreys [1976] E.R. that discretionary power should be exercised only in most exceptional circumstances Laskin, C. J.C.: (dissent) - discretion does exist for trial judges to stay proceedings for abuse of process - compares pleas of autrefois convict/acquit and double jeopardy with abuse of process in broad sense - surveys cases where stay of proceedings for abuse of process may be appropriate remedy (e.g. situation where charge is withdrawn/dismissed for want of prosecution, then re-laid--R. v. Koski) - however, in present case, remedy of stay of proceedings does not apply: "The Appellant is in effect asking the courts to undertake the supervision , through their power to control prosecutions before them, of the operation and efficiency of police departments .... Absent any contention that the delay in apprehending the accused had some ulterior purpose, courts are in no position to tell the police that they did not proceed expeditiously enough with their investigation, and then impose a sanction of a stay when prosecution is initiated. The time lapse between the commission of an offence and the laying of a charge following apprehension of an accused cannot be monitored by courts by fitting investigations into a standard mould..."

R. v. Jewitt (1985.) S.C.C. re-examination of issue in Rourke

FACTS: Accused charged with unlawfully trafficking in a narcotic (marijuana); pleaded not guilty b/c he had been persuaded by an informant to sell drugs to an undercover police officer; jury found unlawful entrapment; court stayed proceedings.

ISSUE: i) does discretionary power of trial judge to stay proceedings for abuse of process exist at common law? ii) does a stay of proceedings equal a judgement or a verdict of acquittal by trial court which can be appealed by the Crown?

HELD: i) S.C.C. adopts Young which stated that the discretionary power does exist, but only in "clearest of cases." ii) a decision to stay proceedings is a question of law which can be appealed just as a verdict of acquittal. Dickson, C.J.C.: - Rourke-discretionary power does not exist at common law, only in exceptional circumstances - Amato (1982)--S.C.C. reconsidered Rourke and concluded that it should be applied narrowly to its facts alone Dickson, CJC. (cont'd): - Young (1984, Ont. C.A.) Dubin, J.A.: -, power of trial court judge to stay proceedings is 'Wsidual discretion" where compelling accused to stand trial would "violate those fundamental principles of justice which underlie the community's sense of fair play and decency, and to prevent the abuse of a court's process through oppressive or vexatious proceedings." - "it is a power of special application which can be exercised only in the clearest of cases." - Connelly v. D.P.P. (1964 - Eng.) Lord Devlin outlined rationale supporting the existence of a judicial discretion to enter a stay of proceedings to control prosecutorial behaviour prejudicial to accused persons

R. v. Keyovski (1988) S.C.C. application of dochine under the Charter

FACTS: Accused's first 2 trials on charges on criminal negligence causing death ended in mis-trail because juries could not agree on verdict; at 3rd trial judge ordered a stay on grounds that it would constitute both abuse of process and a violation of s.7 of the Charter; Crown's appeal was allowed on grounds that the accused had to show prosecutorial misconduct.

ISSUE: Does series of trials constitute per se an abuse of process or must accused show prosecutorial misconduct?

HELD: Accused failed to demonstrate that this was "clearest of cases" of abuse of process which would justify stay. Appeal dismissed. Wilson, J.: - accused must demonstrate misconduct by prosecution - charge is serious - proceedings have not occuppied too much time - accused was not held in custody - 3rd trial possibly stretches limits of community's sense of fair play but does not exceed it - s. 7 of Charter subsumes common law doctrine of abuse of process (Bayda, C.J.S.) - finding of abuse of process automatically equals finding of violation of s.7 - onus question: under s.7 it is sufficient to prove on the balance of probabilities that principles of fundamental justice have been violated, BUT under Jewitt-Young test the onus is clearest of cases - standard of "oppressive" prosecution too high R. v. O'Connor (1996) S.C.C.

FACTS: Bishop charged with 4 counts of sexual offences (incl. 2 rapes) alleged to have occured 25 years earlier; accused appl'd for judicial stay of proceedings because Crown failed to fully disclose requested items (i.e. complainants' full medical, counselling and school records); Crown submitted that full disclosure of these documents would "re-victimize" victims; at first, trial judge dismissed this application on grounds that there wag no evidence of a "grand design to conceal evidence" on the part of the Crown, nor any "deliberate plan to subvert justice"; however, this decision was eventually reversed by the trial court judge because Crown still failed to guarantee full disclosure

ISSUE: Did lack of disclosure constitute abuse of process? Should stay have been granted?

BELD: S.C.C. confirmed that stay of proceedings for W~use of process is only an, appropriate remedy in the "clearest of cases." Court split 6-3 where majority held that non-disclosure of this evidence by Crown co- uld not have violated accused's right to full answer and defence. In this case, remedy of stay of proceedings would have been too drastic. Appeal dismissed. L'Heureux-Dub6, J.: - "clearest of cases" applies where prejudice to an accused's right to make full answer and defence cannot be remedied or where irreparable prejudice would be caused to the integrity of the judicial system if the prosecution were continued there should not be a separation between Charter and common law doctrine of abuse of process because both are concerned with individual rights to trial fairness and the general reputation of the criminal justice system - there is a strong trend of convergence between the two, but the Charter is more flexible because where "clearest of cases" threshold is not met, other "less drastic" remedies than a stay of proceedings are available under s.24(l)

R. v. L.(W.K) (1991) S.C.C.

FACTS: Accused charged in 1987 with 17 counts of sexual assault, gross indecency and assault against to his step-daughter and his 2 daughters re: incidents alleged, to have occured between 1957 and 1985; however, no complaints were made until 1986; trial judge stayed proceedings on grounds that the trial would be contrary to fundamental Justice because of the delay of 30 years.

ISSUE: Should the trial proceedings have been stayed? HELD: S.C.C. unanimously agreed with the B.C.C.A. that trial should NOT have been stayed. Stevenson, J: - staying proceedings based on passage of time would be the equivalent of imposing judicially created limitation period for criminal offences (Ed: in Canada there are no limitation periods in criminal law, except in rare circumstances) - ss. 7 & I I(d) guarantee a fair trial, but this is not automatically undermined by a lengthy delay in bringing charges - delay could actually work in favour of an accused because over time Crown witnesses could become unavailable - in Mills, Lamer, J. (as he then was) wrote that it is not the length of the delay which is relevant but its effect on trial fairness - here, the nature/seriousness of the crime supports the conclusion reached by the Court because non-reporting/delay in reporting is very common in cases of sexual abuse and an imposition of limitation period by the courts through stay of proceedings woW4 hinder prosecution in these types of crimes B. TIME MMATIONS (pp. 10-39)

TRIAL WITHIN A REASONABLE TDIE (Cha_rter, s.11(b)) s.11(b) states that "Any person charged with an offence has the right to be tried within a reasonable time

R v. Askov (1990) S.C.C. what is proper test for s. 1 1(b)? court looked at issue of institutional delay

FACTS: 4 accused were charged with extortion & weapons charges; took 3 years to come to trial

ISSUE: i) what is proper test for s. I I (b)? ii) was s. I I (b) violated in this case?

]HELD: i) factors to be considered in determining infringement of s.11(b): i) length of delay ii) explanation for the delay iii) waiver iv) prejudice to accused ii) Even if delay of one year prior to preliminary hearing could be attributed to adjournments requested by defense counsel, a 2 year delay attributable to lack of facilities in Brampton (Peel district, Ontario) can be said to be so contrary to the rights of the accused under s. I I (b) that the only appropriate remedy is a stay of proceedings. Cory, J. a) U.S. Approach - 6' Amendment guarantees right that "in all Criminal prosecutions, the accused shall enjoy the right to a speedy and public trial." Barker (1972): - guy charged with murder who was not brought to trial until 5 years after the crime was committed - court held that flexible approach needed to be taken in cases involving delay Powell, J noted 3 interests which were protected by the & Amendment: i) prevention of oppressive pre-trial incarceration ii) minimization of accused's anxiety/concern iii) limitation of possibility that defense would be impaired or prejudiced - also noted added dimension of societal interest (e.g. increased financial cost to society of delayed proceedings, negative effect on credibility of justice system, assisting accused in "getting off' - in balancing the rights of the individual against societal interests, 4 factors were identified as a test for the infringement of the right to a "speedy trial": i) length of delay* H) reason for delay iii) accused's assertion of the right iv) prejudice to accused first factor triggers test; i.e. if delay excessive, then other 3 factors must be considered to determine if right has been violated b) Canadian approach post - Charter - purpose of s. I I (b) - aspect of fundamental justice guaranteed under s. 7 - protection of individual's right - inference can be drawn of community/societal interest protected as well under s. I I (b): - collective interest in ensuring criminals are brought to trial - individuals on trial must be treated justly - factors to be considered in determining infringement of s.11(b): i) length of delay - not a threshold requirement like in U.S., but rather one factor to be balanced among others ii) explanation for the delay a) Delay Attributable to Crown - these will be weighed in favour of the accused - complexity of case will be taken into account (i.e. cases which require longer delays than would normally be acceptable b) Systematic or Institutional Delays - lack of institutional resources cannot justify unreasonable postponement of trials - Crown must show institutional delay is justifiable - how long is too long? - must consider geography, population and material resources of province and district - then comparison must be made to the standard maintained by the best comparable jurisdiction in the country looking at appropriate ranges of delay to determine what is a reasonable limit c) Conduct of Accused - can look only at situations where actions of accused directly caused the delay or where acts of the accused are shown to be deliberate and calculated tactics to delay trial - here, burden of proof is on the Crown iii) waiver - waiver of Charter right must be "clear and unequivocable ... with full knowledge of the rights the procedure was enacted to protect and of the effect the waiver will have on those rights in the process - failure of accused to assert the right does not give the Crown a license to proceed with and unfair trial (i.e. silence/lack of objection cannot constitute a lawful waiver) onus on the Crown (on balance of probabilities) to prove that waiver can be inferred iv) prejudice to accused - it should be inferred that a very long and unreasonable delay has prejudiced the accused wdstence of inference of prejudice drawn from a very long delay will preserve the pre-eminent right of the individual - it is open to the Crown to demonstrate that the accused has not been prejudiced c) Application of Test to Facts - discussion of bad state of affairs in Peel district of Ontario - characterizes delay as being "of such an inordinate length that the public confidence in the administration ofjustice must be shaken. Justice so delayed is an affront to the individual, to the community & to the very administration ofjustice. The lack of institutional facilities cannot in this case be accepted as a basis for justifying the delay."

in concurring opinion, Lamer CIC., Wilson and Sopinka, JI, disagreed with the opinion of the majority that s. 11 (b) protected a societal as well as an individual interest because of statement by Majority that "6 to 8 month delay would be the outside limit of what is reasonable" the impact on the Ontario courts was described as "staggering" (e.g. 34,000 charges were stayed, dismissed or withdrawn in one 6 month period) R. v. Morin (1992) S.C.C.

Askov reconsidered.

FACTS: Accused was charged in Jan.'88 with driving with an excess of amount of alcohol; in provincial court (Feb.'88) counsel for the accused requested the "earliest possible trial date'; trial was set for A4ar.'89; at trial, accused brought motion to stay proceedings because the delay of 14 & 1/2months infiinged. on her right to be tried within a reasonable time; motion was dismissed and accused was convicted on the "over 80" charge; on first appeal, court stayed the charge; at Ont. C.A. the conviction was restored.

ISSUE: Was accused deprived of right to be convicted within a reasonable time under s. I I (b)9

HELD: Delay was not unreasonable in this case; accused was not deprived of s. I I (b) right. Sopinka, J.: - while accused has ultimate or legal burden of proof throughout, Crown has secondary or evidentiary burden under circumstances to show that a delay was justified (Smith) - restatement of factors to analyze whether a delay has hiffinged the accused's rights under s. 11 (b) (i.e. "how long is too long?"): i) length of the delay ii) waiver of time periods iii) reasons for the delay, including: a) inherent time requirements of the case b) actions of the accused C) actions of the Crown d) limits on institutional resources e) other reasons for the delay iv) prejudice to the accused factors: (mostly repetition of discussion in Askov) i) length of delay - court must examine period from the charge to the end of trial - charge means date on which information sworn or indictment preferred - inquiry into unreasonable delay triggered by application under s.24(l) - applicant has legal burden of establishing a Charter violation - the inquiry should only be undertaken if the period of delay is of sufficient length to raise an issue of reasonableness ii) waiver of time periods iii) reasons for delay a) inherent time requirements of case - as complexity of case increases so does amount of delay (e.g. whether case must proceed through a preliminary inquiry) b) actions of the accused C) actions of the Crown d) limits on institutional resources - purpose of the suggested period in Askov was not to be treated as a limitation period which was inflexible, but rather to indicate that a delay caused by limited resources can only be tolerated up to a point - comparisons to other jurisdictions should be applied with caution - suggests that in Provincial Courts a reasonable institutional delay would be 8 to 10 months; the range of 6 to 8 months suggested in Askov after committal for trial would not be unreasonable iv) prejudice to the accused application of test to the facts: - most important factor in the case is limits on institutional resources - because of the increase of cases dealt with by Ontario Provincial Court, especially in this high-growth region (Durham), the court allowed a period of systematic delay within the upper range of the guideline: "While one cannot use institutional resources to nullify right to be tried within a reasonable time, one also cannot use rapidly changing local conditions to compel a general amnesty - NB: deviations of several months in either direction can be justified by the presence or absence of prejudice - accused led no evidence of prejudice - could prejudice have been inferred from the delay? - Crown submitted that it sent a letter to defense counsel offering to attempt to obtain an earlier date for trial - while accused was not Muired to do anything to expedite trial, it was considered that the fact that accused did not do anything to expedite her trial was taken into account to assess whether there was prejudice Lamer, CIC. (dissenting): - would allow the appeal and restore the stay - accepts principles and guidelines set out by Sopinka but disagrees with approach to prejudice issue: - understood Cory's reasons in Askov to be that the onus on the applicant in regards to prejudice only exists when the applicant is seeking a remedy in additio to a st Effects of Moyin: 1. More serious the charge less likely it is to be stayed 2. Crown no longer has burden of proving the delay was caused by the accused, that institutional delay was justified, or that accused has waived s.11(b) right 3. Reduces significance of comparative jurisdiction test 4. Increases significance of the prejudice of the accused

In short, s.11(b) challenges post-Motin are less likely to succeed unless the accused demonstrates serious prejudice.

R v. Kalanj (1989) S.C.C. when does time begin to run for purposes of s.11(b)?

FACTS: Accused were arrested without warrant, following a police investigation of allegation of theft; they were released the same day after being told that charges had been laid; 8 months later informations were sworn; Crown explained delay as necessary for the preparation of its case; trial was set for two years after the information had been sworn; at trial, the indictment was quashed on basis that accused's s. I I (b) right had been violated; considered that time of time from swearing information to trial did not constitute an unreasonable delay, but that delay between arrest/release and swearing of information had been unreasonable; B.C.C.A. allowed Crown's appeal and ordered new trial; accused appealed.

ISSUE: i) were accused's rights under s. I I (b) violated? ii) should pre-charge delays be included in the calculation of whether there was unreasonable delay in bringing the accused to trial?

BELD: Accused's rights were not violated because of the pre-charge delay; the period of 8 months prior to charge should not be included because of the unpredictable nature of the length of the "pre-information or investigatory period." McIntyre, J. (L'H-D, and LaF, JJ., concurring): - s. I I is limited to those "charged with an offence", and should not therefore be equated with s. 10; it only refers to aspects of a trial - difficult to predict length of investigatory period prior to the swearing of an information; can only be determined on a case-by-case basis - therefore, time period begins after the information is sworn (departure from earlier cases); only in exceptional cases should pre-charge delay be taken into account because of prejudice - decision of B.C.C.A. should be upheld; appeal dismissed. Lamer, J.(dissenting): - B.C.C.A. effed in categorizing the 8 month delay as a pre-charge delay, and instead it should have been included in the period to determine whether the accused had been tried in the requisite reasonable time period - Appeal should be allowed Wilson, J. (dissenting): - time frame under s. I I (b) should not be determined when the information is sworn, but rather as soon as the "impact of the criminal process is felt by the accused through the service of process upon him in the form of a summons or notice of appearance or an arrest with or without warrant." - therefore, approach to s. I I (b) should be fle)dble to fulfil purpose of the provision

R v. Potvin (1993) S.C.C.

FACTS: Accused was charged with criminal negligence causing death; information sworn Sept.'88; series of delays, accommodating both parties occurred, resulting in the trial date finally being set for Dec.'90; at trial accused applied for stay of proceedings on grounds that his s. I I (b) right had been violated; Crown appealed which was allowed by the Ont. C.A.; the matter was remitted for trial which was held June '92; accused appealed again.

ISSUE: Should appellate delay be included within the time frame for unreasonable delay under s. I I (b)?

BELD: Full bench accepted Morin as authority for s. I I (b) analysis. Majority held that appellate delay could not be considered under s.11(b). Sopinka, J. (L'H-D, Gonthier, Cory and Iacobucci, J.J. concurring) - s. I I (b) does not apply to delay in respect of an appeal from conviction by the accused nor an appeal from an acquittal by the Crown - after an acquittal the person is no longer charged - an appeal is a right open to the accused and Crown; it is not a governmental action - no distinction between an acquittal after trial and a judicial stay - where fairness of trail is affected by delay of appeal proceedings, the better remedy may be found under s. 7 (see R v. L. (W.K.)) - appeal dismissed McLachlin, J. (Lamer, C.J.C. and Major, J. concurring) - rights protected by s. I I (b) are engaged within the period between a verdict or stay and "the final disposition of the criminal charges." Pages 616-660 Dominique Hussey 843-6898

430W W3EK]B XXQr31:30XC;W3W3BZ4W 4313M WMIM XM - JP4303E;tMKALqrXQ0W

Recall there are only two types of proceedings: 1) on indictment 2) on summary conviction

Before a provincial court judge by either way of proceeding, the charge is in the information. Trial on indictment Before aprowncial couitjudgc The information is treated as an indictment (ss. 2 and 554(4)) Before a Superior court The information which commenced the process is replaced by the indictment which becomes the charging instrument in the trial (s. 566 and Form 4)

One indictment or information is allowed per trial, and per accused. Within one indictment, there can be more than one accused, and more than one count (charge) A formal attack can be against one or more counts within an indictment.

N.B., in a summary conviction, formal objections may have the effect of a full defence because the 6 month time limitation on summary offences may bar new proceedings.

A motion to quash based on a defect apparent on the face of an indictment or an information must be brought before the plea, and after requires the leave of the court (s.601).

Cw"3M40UWX)01%/1P003MMffj&X - , Q033%YM4CqrX4:>W1S: 1) Insufficiency 2) Duplicity 3) Improper joinder of counts

3L)o 313txiwixr:E:rLcAL4e3mc,--y: A charge in an indictment not complying with ss. 581 through 583 is insufficient.

S. 581 (1) Each count in an indictment shall apply to a single transaction, and shall contain a statement that the accused commi'tted an indictable offence which is specified. (2) The statement in (1) may be (a) in untechnical and language with no unnecessary elements (b) in the words of the act describing the offence or declaring it to be an offence (c) in words sufficient to give the accused notice of the offence (3) A count shall give enough detail of the circumstance of the offence to give the accused reasonable information wit the act or omission, and to identify the transaction referred to. (4) For offences under s.47, and 49-53, every overt act to be relied upon shall be stated in the indictment. (5) A count may refer to the act that creates it, and this will be takeri into consideration to determine whether a count is sufficient.

S. 583. No count in an indictment is sufficient because of insufficient detail about matters, which in the opinion of the court are outside of s. 581 requirements. No count on an indictment is insufficient by reason ONLY that: it does not name the (a) victim (b) interested parties (c) victim or potential victim of fraud it does not set out (d) writing (e) words which are the subject of the charge it does not specify the means by which the offence was comn-fitted

I it does not name or describe nay person, place or thing with precision it does not have consent of a person or authority whose consent is required.

The same provisions apply to summary conviction proceedings (s. 795)

Policy: These provisions attempt to strike a balance between ensuring the accused gets adequate notice of the charge so he can prepare the defence, and avoid undue technicalities. This used to be to guard against excessive penalties for minor offences.

Brodie v. R [1936] SCC The charge did not give the accused adequate notice, because it did not meet the following test: A charge must specify the time, place and the matter. e.g.: You cannot say, as in this case, that the accused is being charged as being "a party to a seditious conspiracy". The actual agreement must be specified.

R v. McKenzie [ 19721 SCC facts: cab driver was employed on terms allowing him to retain a percentage of daily receipts, the remainder to go to his employer. On a specified date, he picked up 5 passengers, and dropped them off at various destinations. He only recorded one trip for one passenger, and under-reported the fare received by $16.50. He was charged for "committing theft of $16.50". TRIAL: conviction CA: Set aside the conviction. Theft is described in two different sections of the act, and under s. 492(3) is invalid unless it contains an averment of the essential ingredients of the particular section, which was the charge. Since the indictment did not charge s.276(l) [s.3301 a conviction could not rest on it. SCC: Get a life, thief. The charge complied with the code, because there was no genuine doubt as to the conduct to which the charge related. If further particulars were required describing the means by which the offence was alleged to have been committed, they could have been sought under s. 4970)(f).

R. v. Cote [ 19781 SCC Facts: Accused was charged in an information for refusing to comply with a demand by a peace officer to provide a breath sample suitable for analysis contrary to s, 235(2). Sask. CA: the information did not allege the essential element that the refusal was "without reasonable excuse". SCC: Tough luck. No defect. S. 510(5) [581(5)] indicates that when reference is made to sections of the code, consideration will be given to them to determine whether the count is sufficient. Principle: Accused is to be reasonably informed of the transaction so that s/he can make a full defence and have a fair trial. All the facts were stated. It was impossible for the accused to be misled. We can't revert to extreme technicality.

R. v. WI S Devs. Corp. [ 1984] SCC Facts: An information was laid alleging 16 counts of breaches of the Aeronautics Act, and 16 counts of breaches of the regulations of the Act. I I 'I uirby s. Held: Information with 32 counts of breach vo'd ab init'o for want of sufficient detal req ed 581 (3). Every offence under the act is based on the "operation of a commercial air service without the requisite license or certificate". A definition section in that act defined "commercial air service" ym vaguely. The statute under which the information has been laid casts a broad net, and the prohibition was directed at many diverse and unrelated issues of aircraft in Canada, all within the definition of the operation of a commercial air service.

2 R. V. RAQ [ 19901 SCC Facts: each accused charged with committing a sexual assault on a child "between December 2 and December 20", 1985. The child was 7 at the time, and 8 when testifying. The child's testimony appeared to put the date of the assault in late 1984. TRIAL: the testimony of the child's mother and the expert on child abuse could not accurately establish the date off the offence - which is an essential element. The judge refused to amend the information to include the correct time, on the evidence. Acquittal. SCC: The charge was adequate, and should have been amended to accord with the evidence. Wilson, L,-D, Gonth, Cory, McL: Since Brodie: courts have an increased tendency to reject insufficiency arguments because they are overly technical. ::R. 10. t198 be.1ime peticd:s n ".h:~the::Accused:.Was.:.,unlawfuRy stealing Q: SCC~,~J fi unw, . peci 64 d hic ev~C Jime enc !.,~thelvlacelhe victim:.andlhe offence,were,all man sp Nrthplm-~IW- -~p Od ci- at: tit~:rmvl*~i. ,nd 'A fti deso na Of M, "W4, ta lu Up- :t -ribe'lhe:: t~.:q,&Mce.: Th.e t tha diffi 4ft: for:

.&*-LI~1~1.", t kir~ness: wil be.idegt Inv- i i Th,cul. ~the defence ta~prepare,~. its case~ isriot suffi6kiif to A ::diffi and f I y with: in evidence,

Bottom line: What constitutes reasonable or adequate information wit an act or omission to be proven against the accused varies from case to case. Some factual matters permit greater descriptive precision than others. In this case, having regard to the nature of the offence charged and the age of the victim, the information provided was adequate. Time is not an essential element of every offence. Conclusions from the authorities: 1) Time must be specified to provide the accused with reasonable information about the charge to allow fall defence and fair trial. There may be circumstances where greater precision is required. 2) If the time on the information inconsistent with the evidence, and time is not an essential element, or crucial to the defence, the information need not be quashed 3) If there is conflicting evidence wrt the time of the offence, and date cannot be established the information need not be quashed if time is not an essential element or crucial to the defence. 4) If the time of the offence cannot be determined and is an essential element of the offence or crucial to the defence, a conviction cannot be sustained. A court faced with circumstances in which the time cannot be established or cannot be established with precision or information conflicts must ask: 1) is it an essential element of the offence or crucial to the defence. NO? Then no worries. YES? 2) Has the time been proved beyond a reasonable doubt? In this case, irreparable harm is not caused to the accused by amending the information so that the offence of the charge occurred between November 1 1985, and December, 1985. An aside: The standard of the reasonable adult is not appropriate in assessing the credibility of young children. Imprecise details about the time and place do not mean the child does not know who assaulted her and what was done.

Charter s. 11 (d): Any person charged witli an offence has the tight to be informed without unreasonable delay of the specific offence. Decisions to date are cautious.

ReWarren (1985)Ont.HC Facts: Four accused were charged] with offences under s. 163, and have applied for an order in lieu of certiorari quashing the informations laid against them, and an order in lieu of prohibition prohibiting any,judge in the Provincial Court from taking further proceedings in this matter. S. 163 offences are hybrid offences under s. 165, which sets out two alternative modes of prosecution a) an indictable offence and liability to imprisonment for two years or b) an offence punishable on summary conviction. The informations make no reference to s. 165 (a) and (b). When the applicants appeared for the preliminary inquiry, the prosecutor informed them that he intended to proceed summarily, so that the 3 trial, not the P.I. would be held. The applicants applied for an order quashing the informations and preventing the crown from electing to proceed summarily. TRIAL: application disirlissed. S. 11 (a) guarantees the accused has a light to be informed of the acts constituting the offence, but not wrt how the crown plans to proceed whit a charge. The judge granted an adjournment to allow the defence to prepare, since P.I. preparation is different from trial preparation. The crown, had to present their case that same day. ONT HC: The applicants submit that since S. 165 sets out two modes of procedure, two offences are created even though the substantive elements remain the same. The election is part of the specific offence. in W x.Ve I Ri to, the Arftcle:;~, (3)(A) ,0x,.Ufiited:',NA hiternaft It.riant on: C l.and, Bbliti,(W mestic law of Canada.. staes that. det M ~bf any: crt nal charge qm:-' hall Ix.,entitle tolhe followingtrummum In:, the M :tve U 'Of the "i :,a ta da= , An U S w ..Jpc :J~ i, CO- _hAtum''

"'d 11 and:M-,:::C 'd h he I - rd kM16 ta i i 141 0: be., a ofthe dhaitge::i nstlui "Nature and cause do not include mode of procedure to be employed by the crown. S. 11 (a) is to be construed in a manner consistent with art. 14. Therefore, the tight to be informed of the specific ,offence only extends to the substantive offence. Only one offence is set out here. Application dismissed.

Actually just another aspect of sufficiency Common Law: Duplicity prohibits alterriative charges in a single count. Duplicity would be confusing and prejudicial to a fair defence.

Proceedings on indictment: s. 581 (1), s. 5900) Summary Conviction proceedings: S. 7890)(b) but 581 (1) and (6) still apply Remember, motion to quash without leave must be brought before the plea [601 (1)]

Alternative counts are not duplicitous. Alternative charges in a single count are.

v. the Soo (Sault Ste. Marie) f 19781 SCC -R Facts: The City of SSM was charged with discharging or causing to be discharged or permitting to be discharged, or depositing materials into a river and a creek, or on the shore, or the bank, etc., thus possibly impainng water quality. This is an offence punishable on summary conviction for a fine, imprisonment, or both. The offence is set out in S. 320) of the Ontario Water Resources Act. Was this charge duplicitous? N.B.: This was a strict liability offence. SCC: Argument was that the Ontario Water Resource Act charges 3 offences: 1) discharging 2) causing 3) permitting deleterious materials. The principle is well established that if the information in one count charges more than one offence, it is bad for duplicity. Kipp v. A.G. Ont. [ 196 51 SCC. The rule against multiplicity is in s. 7240) of the code: In proceedings to which this Part applies the information ... (b) may charge more than one offence or nelate to more than one matter of complaint, but where more than one offence is charged or the information relates to more than one matter of complaint, each offence or matter of complaint, as the case may be, shall be set out in a separate count. Possible Tests: R. v. Surrey Justices [ 19321 K.B.: if a person may do one without the other, the infon-nation which charged him in the alteniative would be bad. R. v. Madill 119431 Atla CA.: Can evidence be given of distinct acts, committed bythe person charged, constituting two oir more offences? RvAntemat [1972]Ont.CA.: if a section contains two or more. elidrnerits~ it isto be construedas containing only one offence. The essence of the single offence must beable to :e stated with precision. These tests were considered to be too general. Test adopted here: Primary Test must be: Is the accused prejudiced in the preparation of his defence by ambiguity in the [charge?

4 j N

In this case: the charge was not duplicitous. It was not ambiguous, nor uncertain. he city knew the case it had to meet. S. 32 of the Water Resources Act was concerned only with pollution. It was one offence in several modes. The legislature did not intend to create difference offences of polluting.

In Canada, a count is duplicitous not necessarily because it is ol~jectionable on its face, but possibly also because the evidence adduced at trial relates to the commission of more than one offence.

R. v. Hulan [ 19 701 Ont. C.A. Facts: An indictment contained the following counts: that the accused at place X at time Y 1) had sexual intercourse with Faye Hulan, a female not his wife, under age 14, 2)indecently assaulted a female person 3) had sexual intercourse with a foster daughter or female ward of the accused. On appeal against the conviction, it was argued that count 1 was void for uncertainty, because there was evidence of several acts of intercourse in the appropriate period. Held: There are several categories of duplicity: 1) that arising from the fact that one count in an indictment charges more than 1 offence which differ in their nature. The indictment is on its face ol~jecfionable. 2) that where the indictment is not on its face is ol~jectionable., the evidence adduced relates to the commission of more than one offence, any one of which would support a conviction under the count. *** e.g. in this case. S. 4920) is. 5811 and S. 5000)(b), (2), (3), is. 5901 491 (1): Each count in an indictment shall hi eralaprlyta4, i "'gep, ~, , ~ - s ngl&,transaction and ~shall contain and is sufficient if it contains in substance a state =.nfthat:t.he,]a."us6d:0 tted.:an indictable offence,

DL" 1) A: COURE. IS ~ Y101 dvJeCT10naV1e:T)y:.r.e MOMY .1 'aso Tharn - (b) double or multifarious (2) An accused may at any stage of the trial apply to th court4c; amend: brdivide xcount.that (a) charges alternative different matters, acts, omissions.9. that. ~ are, stated in the :alternative in the enactment describinz the offence or declares that the matter&areAN I dictable, ffenceor o (b) is double or multifarious on theground that, as framed, it embarrasses him in his defence, (3) The court may, where ends of justice require it, order a count'46 be::.a m'end'ed or divided into two or more counts and thereupon a formal commencement may be inserte&b.efore,each of the counts into which it is divided. Held: Assuming more than one act of intercourse had taken place, it would still be proper to have charged the successive acts in one count as a single transaction. A number of acts are capable of constituting one offence. Under the 500(3), court has the power to divide, which contemplates that a number of acts can be before the court in one count and that the trial on that count may properly proceed, unless it is ulliust in the eyes of the court. The possibility of a count referring to more than one incident or occurrence is not grounds for holding it to be bad. "Transaction" is not synonymous with incident or occurrence, even if each incident COULD have been the su~ject of a separate count. e.g.: R v. nvnn[I 9551: The accused was charged with theft which occunvd on different days. It was argued that the evidence disclosed more than one offence. It was. held that the theft was one continuous act. The thefts were all from the same owner. Theaccused. could not be convicted of theft again later. Several acts of intercourse can also be treated as one transaction. There was one continuing mens rea for several separate actfis rei. Once the decision had been made by the accused to su~ject his foster daughter to these indecent acts, subsequent acts were simply an extension. E.g.: 2 successive blows to a victim are properly described as one assault. It's the same thing here. There weir even recognized signals which preceded the act including pointing upstairs, etc. Even if this were not the case, s. 491 uses the word "in general" which suggests there are some exceptions to the one transaction rule, and this would be one of them. No pre~judice would be suffered by the accused. In addition, there is a practical advantage to the accused of treating these acts as one

5 ktransaction. It means that when faced with subsequent prosecution, a plea of autrefols convict can be r t eetntered.

R. v. Rafael [ 19 721 Ont. C.A. fa-cts: The accused was charged of unlawfully by deceit, falsehood, or other fraudulent means, defrauding X people of N amounts of money by holding out to them that we would obtain Landed Immigrant status for them in Canada. (s. 3230)). Particulars provided indicated that there were 24 people defrauded. 20 were called as witness to prove that the accused committed fraud in the course of business by charging them a fee for his services. The precise language of his misrepresentation to each of the people differed somewhat -sometimes he stated "I will legalize you" sometimes "I guarantee you will get status", etc. No ol~jection to the charge was made before or after the plea on the basis that it was duplicitous. However, this court held that an o~jection could nevertheless be made, on appeal.. s , 11 ontain ,S. 510(l),.Each.count.lon.ari iiididment,.shallin.,g.&nc)7o!.#pply Ioashig .1nmuction, and ha c and is sufficient.if it contains in substan-ce:a statement that.4heaccused. committed an indictable. offence therein specified. v. Flynn 11955): Many thefts of same owner consideredto be~contiriuous. Autrefois convict would be open if the accused were subsequently charged. R. v. Canavan and Busby [19701: The accused Were charged because: during July 196E~ they.agreed to ~Iace a bet on behalf of another for consideration paid contrary: To ~ s.11 7 7A ~[I 871 The ~ repetition of the offences was held to be ~ comparable ~ to continuing to take,morey.Irom an embezzler, and. therefore they were treated as one transaction. (Same principle as that, recogifized inEulan). In this indictment 24 persons were alleged to be defrauded with different representations. It does not seem that these offences can together be considered to be a single transaction. A separate chargecould have been laid wil each persons alleged to have been defrauded. However, this is not a test that is conclusive. The real test is whether the count itself on the evidence adduced by the crown fairly be said to simply be occurrences within a single transaction? No. Therefore, for failure to conform with s. 510 of the code, the indictment was bad. S. 5290) states that motion to quash an indictment with a defect apparent on its face must be taken before the defence pleads, or after with leave from the trial judge before whorn the trial lakesplace. However courts have not allowed this to stand as an answer to an allegation which turns out to be correct, i.e., that an indictment is vold for failure to comply with s. 510. Q think that this is an incredibly dumb observation. whether the allegation has been found to be true or false, the appeal court will already have considered it, which means the motion has alreadv been made beforc the appeal court, and an investigation performed., contrary to s.5290). If the allegation turns out to be false, the crown would not even tn - to use this as an "answer", because the question would not arise!). It has also been held that duplicity is not a defect apparent on the face of the indictment, so that it falls outside of the ambit of s. 5290) Indictment quashed.

:3> XXJX]PX-43O]P4aX- JC]kfUMCJ4BX- C)I:f 4C4:)1X7UM-tM An indictment/information can contain any number of counts (ss. 591 (1) and 7890)(b)

R. v Clunas [ 19921 SCC Held: 1) There can be joinder of offences or of accused char~ged in separate informations on consent of parties or where offences or accused could initially have beenjointly charged. 2) Summary conviction offences can be joined with indictable offences if the trial is in a provincial court without preliminary inquiry.

Today, it is not necessary for the crown to lay a new information to put separate charges on one piece of paper.

6 S-5900 gives courts wide jurisd ction where it is satisfied that ends of justice require it to order that the count in an indictment be divided. The trial judge has the same discretion to grant a motion to separate the trial of an accused upon one or more counts (s. 691 (3)

In Racco, the court refused to exercise this discretion. R. v. Racco [ 19751 Ont. Co. Ct. Facts: this was an application to sever the counts in an indictment preferred against the accused. The accused was charged with possession of counterfeit money, and in count two, with possession of an explosive substance, at the same time and place, and in both counts without lawful justification or excuse. -520 Anyame number of counts:fbIr any nuniber:~:of indictAble,offences.may"bei joined in the s it.idictrn6nt,lbitt~.the.co.u-iits::~hall.~be distinguished in.the-manner:shown inTorms 3-and 4.

Cou - ;,.tMl I re, us ce~,r CtM dm&,to.: be

tried :se, I ~rnore: It ti -equirtit 'di. =- pontely upomone, or

(4) An orderfor.1fie: sepamte trial of one or:more cou I nts,o in 1: an in 4 i Ct Mentmay be made before or during the trial. Held: In order to determine whether a severance ought to be granted, the facts as expected to be disclosed in evidence am relevant. In this case, there was a nexus between finding the ground burst simulator and the counterfeit US cash. Within minutes of seeing the accused with the ground burst simulator, a search of his residence revealed the counterfeit notes. Greenspan (defence) suggested that even if the court were to direct the jury to keep the two offences separate, they would be unconsciously influenced, so that the accused would be embarrassed in his defence. Furthermore the accused's defence was to be that the money (count 1) and the simulator (count 2) were planted by the police. IF the accused had to defend both counts contemporaneously, he would be embarrassed. 0 would say in both senses of the word). What is the meaning of the words "ends of justice" in s. 520(3)? They embrace the interests of the accused and the interests of the administration of justice generally. The issue is substantially credibility: where there is a close nexus in time and place, it is in the interests ofjustice that both counts be tried together. Greenspan's argument backfired bigtime: ESPECIALLY because both defences would rest on planting by the police, the issue should be resolved by one tribunal for the ends of justice. If the jury has reasonable doubt as to the plant of any one item, they would have reasonable doubt as to the other. A severance would lead to the following undesirable result: if there were doubt wrt the money, the accused would be convicted. If a different conclusion were drawn wrt the simulator, the accused would be convicted. This, even though the defences are the same in both. This is not in the interests of the administration ofJustice. They must be tried together.

02LIVXWG~ X313B,%rXC;3B1B 1) Particulars 2) Amendment 3) Appeal X> 3P8LX--ti4r-XX1VAt-M A court may order the Crown to give the defence particulars under s.587 where an insufficiency attack is launched. An informal undertaking by the Crown to produce the particulars is acceptable, but must be put on record ASAP by order of the court. The crown is bound by the particulars ordered and must prove the particularized charge.

S. 587(3)(c) The trial shall proceed in all respects as if the indictment has been amended to conform with the particular.

Whether particulars are ordered is in the discretion of the trial judge, provided that s/he is satisfied that it is necessary for a fair trial. 15870)]

Function of Particulars:

7 R. v. CGE [ 19741 Ont. H.C. 1) give exact and reasonable information to the accused wrt the charge against him to allow a full

defence 1 12) facilitate administration of justice (e.g. may allow judge to rule on the relevancy of evidence, etc.)

ffic R VAO.:i: s vitiates-, tho char.w _qy:wrtl~-dc~,. in ha bmulgh b~. gnvictio air ~'can-not.:..::remedied~by,.p4rtioUtirs.,: nolonxertme:..~

Thatcher v. R. (1984) Sask. Q.B. Facts: After a P.I., the accused was committed for trial on a charge that he did unlawfully cause the death of a woman, and committed first degree murder. The accused sought an order directing the prosecutor to furnish particulars (pursuant to 516 (1) M) which further described the means by which the accused was alleged to have caused the death. The theory of the crown appeared to have been that the accused did the act that caused the death, but there was also evidence that would allow a jury to conclude that the killing was done by someone else. Thatcher could therefore face the possibility of being found to be a party, and guilty of the offence of murder as an aider or abetter pursuant to s. 21 of the Code. Therefore, the accused argued that he did not know the case he had to meet. S. 516 of the code allows the court to order the prosecution to furnish particulars where necessary for a fair trial. R. v. Lukic . [19461 held that,.,Particulars-~..att,,.~for~th so U$tICC ;i-5AO min halj J p;:and,to Stive. an opportunity to theaccused w1nakt.~XVIUM17 defe Th . -hoWd tot be &Whenthe rsuk n P~Y-,S,Ou'r" M_ would be an injustice in preventing theprosecution -fr6-m:estab1ishft:proof:of an,offence..

R. v. Govedarov (1976) Ont. CA. Facts: the accused were convicting of non-capital murde : ransi ng, ou t:..of,:the.death. oUa restaurant dishwasher during a B and E. The jury was instructed on s 21&the construcfiv.e.::Murdir.pro~ws-lon. The accused appealed based on the failure of the trial judge to order Particulars,:aS, :to. Whickoffences. were enumerated in s.213.

Held: the purpose of requiring particulars was not to provide thi accu I sedwith additional de wit matters in the indictment to inform the accused more fully of the4ct or, Omiisionicharged,agAinst him, but rather to irstrict theprosecution to reliance on apaitonlyof the.,definitfun,of murder~ contained in the criminal code. The true function of particulars is to give,: sufficient information to: enable a defendant to fairly defend himself, without fettering the prosecutorin the:conducf.of thecase. The Crown can rely on any part or parts of the definition of :murder VH le to tht. facts. ~:.Me: Ju can 4P _Pc choose which part is proved. Govedarov and R. v. Khan both appear to be authority for the principle that a court will not order the crown to particularize which underlying or predicate offence it relies on to establish liability for the offence charged. This would nullify the scope of the accused's liability as provided by statute. Held, therefore: the accused is not entitled for the order he seeks. The jury's ability to find him guilty for murder, or for aiding or abetting may not be restricted by an order for particulars.

The CC was amended in 1935 so that a trial Judge has wide discretion to amend an indictment or information [ss. 601, 795, 590(2), (3)].

The following sections are complicated. There are differences in the stages at which the amendment can be made and whether the evidence has to have been adduced and whether the amendment power applies to a defect of substance or form, etc. In general, there is a power of amendment in most cases, but the trial courts are sometimes reluctant to use them. However, courts commonly remedy duplicity by resort to amendment. The crown is asked to choose one of the offences, and the others are deleted. 8 In some of the sufficiency problems. amendment should be considered an option (rather than particulars).

The things being "quashed" or "amended" in this summary of the sections are always and only "indictment or counts therein" unless otherwise provided. 601 (1). A motion to quash shall be made for a defect apparent on its face before the accused has pleaded, or after, only by leave of the court hearing the case in the first instance. The court if it considers it necessary, can order an amendment to cure the defect. (2) A court may amend an indictment or count therein or a particular furnished under s. 587 to make them conform to the evidence if there is a variance between the evidence and (a) a count in the indictment as preferred or (b) (i) as amended, or (b) (ii) as it would have been if it had been amended in conformity with a s. 587 particular. (3) Subject to this section, a court shall at any stage in proceedings amend as necessary where it appears (a) the indictment has been preferred under a different Act of Parliament, (b) that the count or indictment (1) does not state properly one or more elements of the offence (11) does not negative an exception which should be negatived (111) is at all defective in substance, and the matters to be alleged in the proposed amendment are disclosed by the evidence taken on the P.I. or on the trial or (c) that the indictment or a count thereof is in any way defective in form. (4) The court shall, when deciding whether an amendment should be made under (3) consider (a) matters disclosed by the evidence taken on the PI, (b) the evidence taken on the trial, (c) the circumstances of the case (d) whether the accused has been misled or prejudiced in his defence by any variance, error or omission in (2) or (3), and (e) whether considering the merits of the case the proposed amendment can be made without injustice being done. (4.1) A variance between the indictment or a count therein and the evidence taken is not material wrt (a) the time the offence is alleged to have been comn-dtted, if it is proved that the indictment was preferred within the prescribed period of limitation (b) the place where the proceedings are alleged to have arisen if it is proved that it arose within the territorial jurisdiction of the court (5) Where in the opinion of the court the accused has been misled or prejudiced in his defence by a variance, error or omission in an indictment or a count therein, the court may, if an adjournment will cure the prejudice, adjourn the proceedings, and may make an order wrt the payment of costs due to the necessity for amendment. (6) Whether or not to amend is a question of law. (7) An order to amend shall be endorsed on the indictment as part of the record, and the proceedings shall continue as if the indictment or count had originally been preferred as amended. (8) A mistake in the heading of an indictment shall be corrected ASAP, but is not material. (9) The authority of a court to amend does not authorize the court to add to the other acts stated in an indictment for high treason or treason or for an offence against sections 49. 50, 51, 54. (10) "Court means a court, judge, justice, or provincial court judge acting in summary conviction proceedings, or those on indictment. (11) This applies to all proceedings including PI, mutas mutandis.

&g~iny v. Moot-c: Tin-ning theSilver Lining Outward, Like Milton's Cloud judge Russ Barnett, author of this article, disagrees with those angry and ill informed individuals who complain about Can. Cnim. justice system for its habit of letting criminals off on silly technicalities, but admits that they have a point sometimes. In 1988 SCC decided that the conviction or Moore for having been in possession of stolen auto parts could not stand. The crown alleged that in his wrecking business, he acquired parts from stolen vehicles. He then reconstructed trucks, relicensed and sold, and was therefore charged in 76 counts. The information contained the charges of theft (well worded) and (less well worded) charges of possession of stolen property.

9 Problem: no offence was alleged on the information. What he did was indicated, but no offence known in law was disclosed. A new inforniation was sworn. The charges were identical, and Moore entered not-guilty pleas on all counts. The Defence submitted that a plea of autrefois acquit had to stand on the counts because they had previously been held to be a nullity. (The defence had originally argued that the charges had not really been made.) This was a classic case of having one's cake and eating it too. That is: the defence argued that "offences" in the information were not known in law, because they were not specified. When a new information was laid, on the basis of the fact that the wording was in essence the same as that of the dismissed information, he pleaded autrefois acquit. Held: The counts on the first information were bad, and since they were bad, the accused was never in jeopardy. The accused was convicted of some of the charges. On appeal, the autiefois acquit argument was resurrected. The defence contended that one of the counts was not in fact a nullity and could have been amended as necessary, and therefore the appellant was truly in jeopardy. The court of appeal bought this argument, and Moore walked. It went to the SCC, although the accused did not show up. Majority: the appeal was dismissed, although "the Chief Justice and the two wise women who sit upon the SCC" (dissenting) recognized the arguments as nonsense. The ratio of the Moore decision is of course that when an information is quashed, after a plea, for failure to allege a material averment, the Crown cannot later proceed upon a fresh information. This author suggests and laments the fact that the real meaning is that trial judges must not quash criminal charges merely because they are badly drawn up. A charge that gives the accused fair notice of the offence is good enough. It does not consider that the charge must provide sufficient detail to identify the particular act which is charged. S. 510(3) [581 (3)].

R.v.Trembla [1993]SCC Facts: Accused charged with keeping a common bawdy-house for purpose of practicing acts of indecency. [210 (1) 1. After all the defence evidence had been called, the Crown sought to amend - by deleting the words "the practice of indecency" to include "practice of prostitution". Held: This would cause serious prejudice to the accused. S. 601 give the courts wide powers of amendment. Fiinciples: 1) persons accused of a crime must know the chargebrought a ,gainst them in order to present a full answer and defence 2) A court cannot amend where doin ,g so would cause irreparable prejudice. 3) A court cannot amend an information unless the ei4dence tendered can support such a charge. Decision was based on principles 1 and 2. The accused's defence had been that the acts in question were not indecent. The amendment might have been appropriate earlier in the trial.

:3> 4::03MFA,]P]P4Bn1 Once again - motions to quash an information or indictment are to be brought before the defence pleads, and after, only with leave. Appeal courts are reluctant to consider these formal ol~jections for the first time on appeal. There is the fear that the technical ol~jections will be saved for appeal, thereby abusing the process. R. v. Cote [ 19 781 SCC If no objection is made at trial, the matter ends there.

10 Diane Gillies (939-5999) (Pgs 687-700, 813-820, 842-860)

INCLUDED OFFENCES

S.661 -general section on included offences s. 662(l) -a Count in an indictment is divisible (whether offence charged is indictable or summary); -if offence charged includes another offence, accused can be convicted a) of an included offence, despite the fact that the whole offence is not proved or, b) an attempt to commit included offence-, (2) above applies to charge of I st degree murder (i.e. can be convicted of 2nd degree if I-st 011LJ not proved) -subs. 3-6 -application of rule to specific sections of CC -no definition of "included offence"

R j,. Walton (1973 N. B. C. A.) (p. 687)

&Icty: -altercation between drunk brothers, acc stabbed brother 3x, hit him with an axe handle; -acc charged with causing bodily harm with intent to endanger life; -acquitted of the charge, but convicted of assault causing bodily harm-, -acc appealed-argued assault causing bodily harm not an included offence; Lvsue: Is assault causing b.h. included within charge? Held.- -in order to be an included offence, the essential part of the included offence Inust necessarily be "included or involved" in the offence as described in CC or as charged in the count (R. v. Manuel, 1960); -when is an offence aRt included? when the whole off charged can be coininitted without committing another offence, the other offence is not incl.(R v. Carey) -wording of s. 228 implied that endangering the life of a person did not necessarily include assault and -assault not included in charge against accused; -therefore, appeal allowed, verdict: acquittal (Crown did not request a new trial).

R. i~ McDowell (1977 Alta. C. A.) (p. 689)

-accused charged with break and enter, and theft over $200 (a car) -charge read (in part)--"did unlawfully break and enter .... commit(ted) an indictable offence therein, to wit- did unlawfully steal a (car) of a value of more than $200..." -S306(2)(a) (now s. 348(2)) onus on accused to show that s/he did not intend to commit an indictable offence when s/he broke and entered a place Lv.vie: -is "intent to commit" included within "commit"

- ---- cr 11 __ -__ 11 - - -~~11 Held: -3 ways in which An offence may be included 1. Where CC prescribes that certain offences are included offences 2. Where the description of the offence as described in the enactment creating it includes the commission of another offence 3. Where the description of the offence as charged in the count includes the commission of another offence -Moir, JA,- 1. "intent to commit" is not made an included offence by statute (but see note below) 2. essential matter of intent not nec. included in b&e and theft -intent must be present at time of b&e to be guilty of "b&e with intent" -possible to b&e without intent to commit indictable offence, but once in, steal something found in the place, i.e. "b&e and theft" 3. offence as charged does not include "intent" -appeal dismissed Note: 1985 amendment-s. 662(6)-where charged with S348(1) (b) and evidence doesnt prove it but does prove 348(l) (a) (i. e.. intent), acc can be convicted of (a).

Luckett v. R (1980 SCQ (p. 693) Bacts: -acc charged with robbery, convicted of common assault Issue: -is common assault incl. within robbery in CC (undisputed that assault was not incl. within offence as charged) Held: -assault incl. within robbery -acc argued assault not a nec. ingredient in robbery _s. 302 (now s. 343) can be committed by using (threats of) violence towards property, or by stealing from a person while armed with an offensive weapon or imitation--neither involves an assault -thus, common assault not incl. when robbery charge doesn't refer to a particular subs. of s.302 -CA agreed with acc. SCC -rejects above; fact that robbery may be committed without an assault does not alter fact that robbery, as enacted, does include assault -unreasonable to hold that lesser offence must be included in every subs. -lesser offence must be included in offence charged as described in the enactment, but it is not nec. that lesser offence be in all subsections

R. v. Morton (1975 N. S. C. A.) (p. 698) Iss-ue: -does offence of having care and control of a motor vehicle while impaired nec. incl. impaired driving Held: -imp. driving is incl. within having care and control of motor vehicle while impaired -CC does not specify that an incl. offence must necessarily be of lesser gravity in terms of penalty -in other words, it is the ingredients of an offence, not its gravity, that determine whether or not it is incl. in another offence -i.e. when deciding if off. A is incl. within off. 13, it doesn't matter whether A is as grave as B; all that matters is that when B is committed, it is nec to commit A -s. 589 (now s.662) applies in the case of summary conv. proceedings (note: this is exlylicif in s. 662)

JURY TRIALS

(a)Closing Address to the Jury Boucher v. R. (195 5 SCC) (p. 813) Facts: -in address to jury, Crown counsel stated 1. the Crown does not try to convict innocent people 2. if, in the course of investigation, the Cr. has reas. doubts of the acc's guilt, it is Cr's duty to say so 3. he personally believed acc was guilty Held. -error in law occurred -improper for either counsel to express own opinion of guilt or innocence -grossly improper for Cr. counsel to suggest that the jury should find the ace guilty because the Cr. had made a careful examination such that they were satisfied of ace's guilt -new trial ordered

PisanL v. R (1971 SCC) (p. 815) Bacts: -appeal on conviction on charge of possession of counterfeit money -Cr. counsel, in address to jury, made statements such as: acc. knew fingerprints couldn't be lifted from fake bills acc. was associating with distributors, pushers of fake $$ -judge did not act on defence counsel's move for a mistrial, but proceeded to charge the jury Issues: -should mistrial have been granted on basis of Cr. counsel's address to the jury? -was that address of a nature such that it deprived the acc of a fair trial, resulting in a miscarriage ofjustice? Held.- -Cr. counsel's address to jury included statements of facts to be considered for conviction for which there was no evidence to support, rather, they came from Cr. counsel's personal experience or observation -the "standard" charge by the judge was insufficient in this case, judge did not try to erase the effects of the address -nothing in defence counsel's address can be regarded as justifying Cr's address -no unyielding general rule that an improper address to jury is per se conclusive of the fact that there has been an unfair trial and that a conviction can't stand -must assess these cases on a case by case basis -acc. denied right to fair trial, new trial ordered

Morgentaler v. R (1988 SCC) (p. 818) 1"Clav: -in address to jury, defence counsel stated if the jury didn't like the law, they should not apply it; this would send a message to Parl. that law should be changed Held.- -such statements are improper -long-settled in Anglo-Can crim. law that it is judge's role to state law, jury's role to apply that law to facts of the case -jury members are not experts on law; must be guided -def. counsel's reasoning could lead to gross inequities e.g.. a racist jury could be told to acquit a white man accused of killing a black PIAA

-it is true that juries have a defacto power to disregard the law as stated by judge Oury is never asked to give reasons) -but recognizing this reality is a far cry from suggesting that counsel may encourage a jury to ignore a law or to tell the jury it has the right to do so

(b)Jury Unanimity X v. Littlejohn (1978 Ont. CA) (p, 842) I'llav: -acc and 3 others charged with conspiracy to import a narcotic -while jury was deliberating, they sent a note to the judge: "The problem is one of us knows of guilt but cannot bring one's self to say it outright to make it unanimous. Please advise us." -over the next 6hrs the judge made various comments in response to the first and further notes from the jury such as: -he explicitly stated "everyone has seen (the note)"; (no one dissented from this) -he reminded them more than once of the oath taken to reach a "true verdict" -duties, even distasteful ones, must be faced--further reference to oath -his reluctance to accept a hung jury; did not believe they were really deadlocked -his belief that "someone may be evading his or her duty, not paying attention to the oath" and "someone is not following the oath that was taken" -if he thought "for one moment" they were really deadlocked, he would dismiss them -the jury left the courtroom after the last meeting with the judge at I 1:46pm and returned at 11: 5 5pm with a guilty verdict I~V,Vue: -did judge's statements to jury during deliberation constitute an interference with their freedom of deliberation, i.e. were they coercive

- I - I

- I - I I/e/d'. -judge's statements not an interference with jury's freedom of deliberation -well est. that judge must avoid lang. which is coercive, which constitutes ail interference with the right of a jury to (dis)agree free from extraneous influence/pressure (R. v. Alckenna) -must avoid conveying to juror that s/he should agree with the majority despite genuine doubts (R. v. Davey) -first note is critical to determination ifjudge crossed the line -note meant that all 12 were satisfied of guilt but one was unwilling to record it -judge dealt with this in an impeccable manner -although true that the jury didn't deliberate for a long time after final instructions, this is not the determining factor (in response to acc's distinction bet. this case and Shoukatallie v. The Queen, a Denning decision in which he referred to the fact that the jury didn't seem to be coerced by the judge because they deliberated for some time afterwards) -appeal dismissed

Law Reforin Commission, Working Paller-- The Jury in Criminal Trials Unanimity Requirement (p. 847) Recommendation I -requirement of unanimity should continue to be an essential characteristic of the jury COmment - unan. requirement has ancient history -Maj. verdicts now allowed in certain cases in 6+ U.S. states, 4 Aust. states, Eng., Scotland, and Trinidad and Tobago Arpmenty inAvour afazoi. verdicty 1. Problem of Hung Jury avsimption: mc#. verdicts=feiver hungfitries --- > lime and $$ 'Vaved -in 1972, only .009% of persons charged with indictable offence retried because of a hj. -1976-77, 1.02% ofjury cases resulted in hungjury, contrast U.S.--5 to 5.5% -relaxing requirement would not eliminate hj. -ecoti. arg. inconsequential 2. Problem of Corrupt Juror assunil.vion: usually the case that I or 2 unreasonable or corrup1jurors hold outand hang ajury that would otherivisv have reached a verdict -in Can, no compelling evid. that corrupt/obstinate jurors pose a serious threat; hj. not concentrated in cases where jury intimidation likely -U.S. stats show jury deadlock results only where a substantial minority viewpt present in first polling ofjury; -first ballot should be seen as measure of ambiguity of case; follows that case itself must be primary cause of a hj. -even if corrupt jurors a problem, more careful screening is better sol'n 3. Unanimous Verdicts are Anomalous aysIIIIII)tion: it. )'~ are anomalou.8 when conyiared to decisioti-makiiig rulesfor other democratic hislitutions; legislative boches, al.)I)ellale cis etc. decide on basis on someform (tf maj. vote -important differences bet. jury decision-making and groups mentioned in analogy 1. acc not convicted unless jury satisfied of guilt beyond reas. doubt 2. jury has little time, only info it can use is that presented to them 3. indiv. jurors are unskilled at evaluating litigious evid., it is collective experience and deliberative process that results in accurate fact-finding 4. jurors determine essentially factual questions, while most other tribunals also deal with questions of law and policy or both 4. Unanimity Rule a Sham assunil.vion: matiy verdicts are result of compromise or acquiescetice by a millority because of coalitioti or verbalpress-ure -compromise verdicts may not be undesirable--strength may be interaction bet. jurors -compromise may still be needed to reach maj. verdict -if unan. req. has important benefits, that fact that some may acquiesce is not a serious cost jriaimelas iti hivout - a) Uitatiitni& -most fundamental rule is "proof of guilt beyond a reas. doubt"--how can this be fulfilled as long as some of the judges doubt (Sir James Stephen) -2 runctions: 1. eliminates to greatest extent that innocent person will be convicted because of error in evaluation of evid. 2. ensures moral acceptability of convictions because public is not left in doubt as to whether innocent persons are being convicted 1. Incr. Accuracy of Fact-Finding cissuml,vioti: unam req. reduces the risk that hniocent people will be cotivicted by hicr. accuracy of.juryfitcl- finding -research shows under unan. rule, minority views more likely to be expressed, considered and superior quality of discussion 2. More Acceptable Verdicts assumption: unan. rule leads to more acceptable verdicts thall maj. verdicts do (a)Can. jurors support unan. rule -proof beyond reas. doubt has important symbolic value (b)public generally aware of rule and few oppose it in all cases (in study, 3.7% opposed in all cases, 33. 1 % support rule in all cases) -appears that for offences presently tried by jury, great maj. of Cans. in favour of unan. v. (c)acc more likely to accept verdict, knowing unan. required (d)unan. rule brings community standards to bear on process, minority groups more likely to be heard Thatcher v. R (198 7 SCC) (p. 856) RICI~V: -acc charged with "unlawfully caus(ing) the death of JoAnn Kay Wilson and did thereby commit first deg. murder." -all the jurors were convinced that he was not innocent -dispute existed over nature of guilt; some believed he personally killed her; others believed he aided and abetted someone else to do it Lvsue: -rnust nature of acc's participation be proved to each and every juror before s/he can be convicted Held.- -Cr. need not specify whether acc was charged as a party or principal; jury unanimity not required as to the particular nature of the participation -true that the two theories are factually inconsistent, but they are not legally different -reasoning which would lead to acc's acquittal would, for example, support acquitting x and y of killing z even if evid. showed that when x and y entered A house, z was alive, when they left he was dead; each says the other murdered z but admitted to aiding and abetting; ridiculous to say both should be acquitted ifjury disagrees as to who actually killed z

R v. Lasprence (1987 Ont. HC) (p. 858) h'tICIS: -sentencing hearing of L. -baby shaken to death -L. convicted of manslaughter (originally charged with 2nd deg) -jury had two bases for mansl. conviction: either L. killed baby or c/l wife killed baby and L. criminally negi. in failing to prevent her -judge stated that k was convinced that L shook baby to death Commentary by Delisle & Stuart: -judge's act (stating his conviction of Us guilt) was clearly the july_!Sjob, as acc has right to have jury assess the facts (s. I I (f) of the Charter) -judge did not ask for a special verdict defn: a verdict in which judge goes beyond the general verdict (simple

99 guilty" or "not guilty"), asks jury basis upon which it was reached -special verdicts allow judge to determine the proper punishment -but, altho' they are allowed in crim. law, they have been condemned recently by Eng. and Can. jurisprudence (as well, fell into disrepute in the 19th C.) -a special verdict can raise more difficulties that it solves because jury may have reached the verdict on different grounds -jury is entitled to arrive at a unan. verdictf.or different reasons

Pletise ecd/ if something is unclear or ifyou have ciny questions.

I- I - F__ Tripta Sood 985-9255 p.738-805

TRIAL PROCESS: JURY TRIALS

In General: -except for s.553 offenses, the accused can elect the mode of trial -jury trials: governed by: 1. Provincial Jury Act (statute) to govern initial jury pool 2. Federal statutes governing selection of jury from pool -challenges: 1. Peremptory: not required to give reason for rejection of juror 2. For Cause: must challenge ability to fulfill function of a juror

R. v Biddle, 1993 Ont. C.A.

Facts: -appeal by accused, convicted of two counts of assault causing bodily harm, and two counts of choking with intent to commit an indictable offence (acquitted on two others) -jury selection: Crown: 4 peremptory challenges, 48 stand asides Defense: 12 peremptory challenges, no stand asides -as a result, accused was tried by an all-female jury Conflict: -accused argued reasonable apprehension of bias on the part of the Crown, in choosing only female jurors, to help its own case (ie, female jurors would be more sympathetic to the women attacked) Held: -no reasonable apprehension of bias found; accused appeal rejected Reasoning: -R. v Bain, Cory J. : accused must demonstrate abuse of the stand aside provisions by the Crown, in order to prove reasonable apprehension of bias -Cory J. meant that the power must be misused, and that the appellant must show negative consequences on the impartiality of the jury as a result of the misuse -it is not enough to show that the Crown simply chose the jury that would work best for them -in this case, "the fact that a jury is unisexual does not establish a reasonable apprehension of bias" -the fact that women may identify more strongly with the victims does not equal bias, because any reasonable person would sympathize with them -in addition, the case was not concerned with the honesty of the victims; solely a question of identification of the accused R. v Parks, 1993 Ont. C.A.

Facts: -accused charged with second degree murder; convicted of manslaughter by a jury, and sentenced to seven years by trial judge -accused, a drug dealer, was given money by the deceased for cocaine -when accused failed to produce drugs, deceased grabbed him and demanded his money -in front of several witnesses, accused was stabbed in the hand, then stabbed the deceased in the heart during a struggle Conflict: -at trial, defence counsel indicated that he intended to challenge prospective jurors for cause, and wanted to ask two questions to jurors: 1. Will your ability to judge impartially be affected by the fact that drugs are involved? 2. Will your ability to judge impartially be affected by the fact that the accused is a black man and the deceased is a white man? -trial judge refused to allow either question, and accused argues on appeal that the questions should be allowed Held: -question #1 is not allowed; question #2 is allowed Reasoning: 1. Question #1 -implies that a juror should not consider the issue of drugs in judging witnesses, when, in reality, this is relevant to the case at bar 2. Question #2 -trial judge stated that juries should not be questioned, relying on the presumption that duly chosen and sworn jurors can be relied on to do their duty and decide the case without prejudice: this presumption is valid, but.... -question was asked only to determine fitness as a juror, not to challenge jurors' beliefs -was not an attempt to obtain a favourable (biased) jury for accused -simply an attempt to prevent bias from destroying impartiality Relevance of the Question Posed: Nuestion must address both attitudes and behaviour flowing from those attitudes *partiality = bias only if it will impact on the decision reached by the juror *therefore, the issue is, if prejudice existed, it would cause that juror to discriminate against the black accused in arriving at the verdict -the question dealt with both attitudes and behaviour, which would influence the juror's decision, therefore, the question is relevant Does the question fulfill the Sherrall test? *was there a realistic possibility that jurors would,due to racial prejudice, not be impartial? *was there a realistic possibility that a juror would be biased against a black accused charged with murder of a white person? *was there realistic possibility that a prospective juror would be influenced in the performance ofjudicial duties by racial bias? *the answer involves a balance betwe6n the presumption ofjury impartiality and the threat of a verdict tainted by racial bias -evidence in studies of racism in Metro Toronto; racism is part of society -there is a realistic possibility of racial bias in some trials Cost-Benefit Analysis: -very little cost, except in the increased time required in jury selection -no embarrassment to juror; no intrusion into privacy -Benefits: 1. Eliminate potentially biased jurors 2. Make jurors aware of race issues and potential bias 3. Enhances the appearance of fairness to accused

R. v Biddle, 1995 S.C.C.

Facts: -see above; appeal by accused on an issue of procedure, for which a new trial was ordered by Sopinka J. for the majority -the issue of reasonable apprehension of bias was dealt with by three judges Reasoning: Gonthier J.: -Crown responsibility to ensure fairness of trial -injury selection, this means that the Crown must select a jury which is impartial, representative, and competent-best qualified to judge the case -representativeness is key, because it furthers the perception of impartiality, even if it is not a guarantee of impartiality, therefore, it must be sought after McLachlin J: -aJury must be impartial and competenL no requirement of representativeness -for centuries, all-male juries were considered impartial; why not all-female? -representativeness sets an impossible standard: which groups must be represented? to what extent? why choose one group and not another? .... etc. -no evidence that the jury selection helped the Crown case, or was partial -no reasonable person would reasonably apprehend bias L'Heureux-Dube I -agreed with McLachlin I 3. Time limitations

(a) By Statute In the case of indictable offences there is only one time limitation now to be found in the Criminal Code. This relates to a three year time bar from the time of commission of the crime of treason, s.48(l).

In the case of Provincial offences, there are usually specified limitation periods. The limitation will not apply if the offence is a "continuing" one.

"A continuing offence is an offence where the conjunction of the actus reus and the mens. rea, which makes the offence complete, does not, as well, terminate the offence. The conjunction of the two essential elements for the comrnission of the offfence continues and the accused remains in what might be described as a state of criminality while the offence continues".

(b) Trial w/in reasonable Time (Charter s.11(b)). 11. Any person charged w/ an offence has the right... (b) to be tried w/in a reasonable time.

R. v. Askov [1990] 2 S.C.R. 1199.

Even if a delay of one year prior to the preliminary could be discounted as being in large part attributable to defence adjournments, the delay of almost two years between the preliminary and the trial had been caused by lack of facilities in Brampton, in the Ont. Judicial District of Peel. Breach of s. 11 and charges had to be stayed.

Cory J.

S. 1 lb) focuses on the individual interest of liberty and security of the person. (Aspect of fandamental justice guaranteed by s. 7 of the Charter.

-Collective interest in ensuring that those who transgress the law are brought to trial and dealt w/ according to the law. -Individuals on trial must be treated fairly and justly. -A trial held w/in reasonable time must benefit the individual accused, as the prejudice which results from criminal proceedings is bound to be minimized. -Erosion of witnesses' memories w/ the passage of time and erosion of the witnesses themselves (!) -Victims themselves may be devastated by criminal acts. They have a special interest and good reason to expect the criminal trials take place w/in a reasonable time. Furthermore, all members of the community are entitled to see that the justice system works fairly, efficiently and w/ reasonable dispatch. The trial not only resolves the guilt or innocence of the individual but acts as a reassurance to the community that serious crimes are investigated and that those implicated are brought to trial and dealt w/ according to law. -hnplicit support for the concept that there is a societal aspect to s. 1 1b) can be derived from the observation that the last thing that some wish for is a speedy trial. There is no doubt that many accused earnestly hope that the memory of a witness will fail and that other witnesses will become unavailable.

Factors in determining an infiingement on s. 11 (b) i) Length of the del The longer the delay, the more difficult it should be for a court to excuse it. This is a factor to be balanced along w/ others. However, very lenghty delays may be such that they cannot be justified for any reason. ii) Explanation for the delay

Smith case a) Conduct of the Crown (or delay attributable to the Crown) Theses are the potential factors that flow from the nature of the case, the conduct of the Crown, including officers of the state, and the inherent time requirements of the case. These delays will weigh in favour of the accused. b) Systemic or Institutional delays A careful and sensitive balancing will be reuqired in order to properly asses the significance of this aspect of delay. From a societal point of view: s. 1 1b) applies to all Canadians in every part of our land. Institutional problems may vary from province to province. Similarly situated communities can provide a rough comparison and some guidance as to what time period constitutes an unreasonable delay of the trial of an accused person. The right guaranteed by s. 1 lb) is of such fundamental importance that the lack of institutional ressources cannot be employed to justify a continuing unreasonable delay. How long is too long? This question must be answered in light of the particular facts of each case. An inquiry into what is reasonable in any region should not be taken in isolation and must of necessity involve a comparison w/ other jurisdictions. it should look at the appropriate ranges of delay to determine what is a reasonable delay. In all cases it is incumbent on the Crown to show that the institutional delay in question is justifiable.

I c) The conduct of the accused (or Delay attributable to the accused) : The right to be tried w/in a reasonable time is an aspect of faildamental justice protected by s. 7 of the Charter. It follows that any inquiry into the conduct of the accused should in no way absolve the Crown from its responsibility to bring the accused to trial. An inquiry into the actions of the actions of the accused should be restricted to discovering those situations where either the accused!s acts directly cause the delay or the acts of the accused are shown to be a deliberate and calculated tactic employed to delay the trial. The burden of proving that the direct acts of the accused caused the delay must fall upon the Crown. Except for those cases where the effects of the accused!s actions are so clear and readily apparent that the intent of the accused to cause a delay is the inference that must be drawn from the record of his/her actions. iii) Waiver The accused should not be required to assert the explicitly-protected individual right to trial w/in a reasonable time. Any waiver of a Charter right must be "clear and unequivocal ... w/ full knowledge of the rights the procedure was enacted to protect and of the effect the waiver will have on those rights in the process". There must be something in the conduct of the accused that is sufficient to give rise to an inference that the accused has undrstood that he/she had a s. 11 (b) guarantee, understood its nature and has waived the right provided by that guarantee. The onus rests upon the Crown to establish on the balance of probabilities that the actions of the accused constitute a waiver of his/her rights. iv) Prejudice to the accused In Smith where a very long and unreasonable delay has prejudiced the accused, Sopinka I states : Having found that the delay is substantially longer than can be justified on any acceptable basis, it would be difficult indeed to conclude that the appellarif s s. 11 (b) rights have not been violated bc the appellant has suffered no prejudice.

But, it will be opened to the Crown to attempt to demonstrate that the accused has not been prejudiced.

The delay is such that it is impossible to come to any other conclusion than that the s. 11 (b) Charter lights of the accused have been infringed. As well, the societal interest in ensuring that these accused be brought to trial w/in reasonable time has been grossly offended and denigrated.

As a consequence of this, a stay of proceedings must be directed. This is unfortunate and regrettable but the trial can be undertaken only if the Charter right to trial w/in a reasonable time has not been infringed. R. v. Kalanj [1989] 1 S.C.R. 1594

Facts

Allegation of theft, accused were arrested w/out warrant. Released the same day after being told charges would be laid. Information was sworn some 8 months later. Trial was set to commence 2 years later.

Trial j. quashed the indictment on the basis that delay violated the accused's right to be tried w/in reasonable time 11 (b) Charter. Delay of 8 months had been unreasonable.

B.C. CA allowed the appeal by the Crown, holding that trial j. had erred in considering the pre-information delay.

Held Appeals were disimissed. Post charge delays were not such that they could deprive the appellants of trial w/in a reasonable time. Trial j. made an error in considering that the pre-charge delay of 8 months was unreasonable and infiinged s. 11.

Analysis McIntyre J. (UH.-D., LaForest concurring)

Whether the rights of the appellants under s. 11 b) of the Charter "to be tried w/in reasonable time" have been infiinged?

Whether pre-charge delays should be included in the calculation of whether there was an unreasonable delay in bringing appellants to trial?

The purpose of s. 11 is to afford protection for the liberty and security interests of persons accused of crimes -after an accused is charged wi an offence. It is concerned w/ post-information period.

The length of the pre-information or investigatory period is wholly unpredictable.No reasonable assessment of what is a reasonable time can be readily made. Delays which occur at the pre-charge stage are not immune from the law outside the scope of 11 (b). -Crim'l Code ss.650(3) & 802(l) protects the fight to make full answer and defence should it be prejudiced by pre-charge delay. -S.505 prompt swearing of an info. Doctrine of abuse of process. -Broad reading of s. 7

Lamer J (dissenting)

The appellants suffered great problems after their arrest. In sucha small community they were the "talk of the town". They suffered w/out a doubt a breach of their liberty as well as a restraint of their security.

The 8 month delay bet. the arrest and the "formal charge" were in violation of s. 11 (B). Crown waited 8 months bc they were not ready to charge and therefore should not have arrested.

The Charter is there to protect the citizen from this kind of situation and we would be remiss if we were to put the victim of such conduct beyond the shield of the Charter while protecting those who are in need of its protection bc police proeceeded lawfully.

Wflson J (dissenting)

The time where the appellants right to a fair trial starts to run is when the impact of the criminal process is felt by the accused through the service of process upon him in the form of a summons of notice of appearance or an arrest w/ or w/out a warrant.

The "talk of the town" analysis should not be considered in assessing the reasonableness of the delay. R. v. Jewitt [1985] 47 C.R.

Facts Jewitt was charged w/ unlawfully trafficking in narcotics. He pleaded not guilty. He admitted selling a pound of marijuana but said he was persuaded to sell to the undercover police officer by a fellow employee who was a police informer. Jury found there had been unlawfid entrapment and the ct directed the clerk of the ct to make an entry on the record staying the proceedings on the indictment.

Dickson CJC i) at c/l, does a discretionary power to stay proceedings in a crim'l case for abuse of power exist as a means of controlling prosecution behaviour which operates prejudicially to accused persons? ii) whether a judicially entered stay of proceedings is a "judgment or verdict of acquittal of a trial court" from which the Crown may appeal to the C.A. under s.605(l)(a) of Crim'l code.

Inherent Jurisdiction of a superiour court to stay proceedings which are an abuse of process was recognized in Can as early as 1886. In recent years though, uncertainty has clouded the question.

In Rourke, the majority affirms an exceptional jurisdiction to stay proceedings whereas Laskin CJC for the minority finds the doctrine of abuse of process a wide-ranging terchnique for the control by the crirdl ct of crim! I procedure in the protection of the processes of that ct. In that case, the initiating process was valid and the only issue was a delay prejudicial to the accused. It is different from this case where the executive action leading to the charge and its prosecution is offensive to the principles in which the administration of jus is conducted by the cts.

Takes the position in Young that there is a residual discretion in a trial ct judge to stay proceedings where compelling an accused to stand trial would violate those fundamental principles of jus which underlie the comunity's sense of fair play and decency and to prevent the abuse of a cts process through oppressive or vexatious proceedings. This is a power that can be exercised only in the "clearest of cases".

Re the verdict of acquittal It would be an anomolous and absurd result if dismissal of the charges on the basis that the proceedings constituted an abuse of process would permit an appeal but a stay of proceedings on the basis that they constituted an abuse of process would not. Administration of jus would be better served by a determination that a stay of proceedings is tantamount to a judgment or verdict of acquittal and subject to appeal by the Crown s.605(l)(a). R. v. Keyowski [ 1988] 1 S CR 657.

Facts Accused was charged w/ crim'l negligence causing death. Hs first two trials resulted in juries failing to agree on a verdict. At the third trial, the trial judge ordered a stay on the grounds that it would constitute both an abuse of process and violation of s.7 Charter. Crown!s appeal was allowed and a new trial ordered. Majority held that it was necessary for the accused to show prosecutorial misconduct.

Wilson I Could a series of trials constitute an abuse of process or whether it is necessary for the accused to show prosecutorial misconduct.

Prosecutorial conduct and improper motivation are but two of many factors to be taken into account when a ct is called

Appellant has failed to demonstrate that this is one of the "clearest of cases " which would justify a stay. -proceedings have not occupied undue amount of time -accused has not been held in custody -trauma and stigma suffered by being an accused is not distinguishable than for any other accused. A third trial does not exceed the community's sense of fair play.

C/I doctrine of abuse of process was now subsumed in s.7.

Appeal dismised. i R v. L. (W.K.) [1991] 1 SCR 1091

Facts Accused was charged in 1987 w/ 17 counts of sexual assault, gross indecency, and assault relating to his stepdaughter and two daughters. I st incident in 1957 and the last in 1985. Victims first complained to the police in 1986. Trial j. stayed the proceedings on the basis that a trial now would be contrary to principles of ftmd'I jus and the 30 yrs delay was "ludicrous" and "specious". B.C.CA said the trial should not have been stayed.

Stevenson I (for the full court)

Does the Charter now insulate accused persons from prosecution solely on the basis of the time that has passed bet. the Commission of the offence and the laying of the charge? NO. In Can. except in rare circumstances, there are no limitation periods in crim`l law.

Pre-charge delay is relevant under ss. 7 and 1 I(d) bc it is not the lenghth of the delay which matters but rather the effect of that delay upon the fairness of the trial.

If proceedings were to be stayed based solely on the passage of time bw the abuse and the charge, victims would be required to report incidents before they were psychologically prepared for the consequences of that reporting.

This was not a case for a stay. R. v. Morin [1992] 1 S.C.R. 771.

Facts Accused was charged w/ driving intoxicated on Jan. 9th 1988. Feb. 23 her cousel expressly stated the earliest possible trial date. Trial set for March 89. On this day she brought a motion to stay the proceedings bc 14.5 months infiinged her right to be tried w/in a reasonable time. Motion was dismissed and accused was convicted.

On appeal the summary conviction appeal ct stayed the charge. Ont. CA allowed the Crown's appeal and restored the conviction.

Held No evidence of prejudice from the accused Delay was not unreasonable. Do not have to resort to the burden of proof analysis.

Analysis Sopinka J (LaForest, Stevenson, lacobucci concurring)

Right to be tried Win a reasonable time

S.11 (b) Any person charged w/ an offence has the right to be tried Win a reasonable time.

The Primary purpose of this s. is to protect individual rights of the accused:

1) Right to security of the person -minimize the sfigrna, anxiety, concern of exposure to crim'l proceedings. 2) Right to liberty -minimixe exposure to the restrictions on liberty which result from pre-trial incarceration and restrictive bail conditions. 3) Right to fair trial -ensure that proceedings take place while evidence is available and fresh.

The Secondary societal interest -society as a whole has an interest in seeing that the least fortunate of its citizens who are accused of crimes are treated humanely and fairly.

-societal interest that is by its very nature adverse to the interests of the accused In Conway, the interests of society must be balanced by the interests of society in law enforcement. In Askov, there is a collective interest in ensuring that those who transgress the law are brought to trial and dealt w/ according to law. Approach to Unreasonable Delay - The factors

A judicial determination balancing the interests which the section is designed to protect against factors which either inevitably lead to delay or are otherwise the cause of delay.

Accepted factors in analyzing how long is too long?

1. Length of delay 2.Waiver of time periods 3.Reasons for the delay a) inherent time requirements of the case b) actions of the accused c) actions of the Crown d) limits on institutional resources e) other reasons for delay 4. Prejudice to the accused.

The role in the burden of proof in this balancing process was set out in Smith

1. Length of delay Examine the period from the charge (date on which an information is sworn or an indictment is preferred) to the end of trial.

Inquiry into unreasonable delay is triggered by an application under s. 24(l) of the Charter. AMlicant has the legal burden of establishing a Charter violation. The inquiry should only take place if the period is of sufficient length to raise an issue as to its reasonableness. (If length is not unexceptional, no inquiry is warranted).

2. Waiver of time periods If length of delay warrants an inquiry into the reasons for delay, it appears logical to deal w/ any allegation of waiver. In order for an accused to waive his rights under I 1b) such waiver must be clear and unequivocal w/ full knowledge of the rights the procedure was enacted to protect and of the effect that waiver will have on those rights. Waiver can be gMlicit or iMplici . 3. Reasons for the delay Delay is inevitable ! a)Inherent time requirements -offences have time requirements which lead to delay. -complexities of the trial lead to delay 2intake requirements" : retention of counsel, bail hearings, police and administration paperwork, etc. -whether a case must proceed through a preliminary inquiry (two-stage process) b)Actions of the accused -motions for a change of venue -attacks on wiretap packets, adjournments, attacks on search warrants. c)Actions of the Crown -failure or delay in disclosure d)Limits on Institutional resources -most common source of delay and most difficult to reconcile. -period that starts to run when the parties are ready fro trial but the system cannot accomodate them. -limited ressources esp. in regions. -follow guidelines

4. Prejudice to the accused -It is the duty of the Crown to bring the accused to trial. -in many cases, an accused is not interested in a speedy trial and that delay works to the advantage of the accused. -s. 1 lb) is a right which can be transformed form a protective shield to an offensive weapon. -may rely on evidence to show prejudice.

Lamer CJC (dissenting) Allow the appeal. Restore the stay of the summary conviction ct.

Both McLachlin and Sopinka placed the onus on the accused to prove prejudice. In applying Askov to the facts of this case -leaving the onus on the Crown has shown that the accused!s liberty and fair trial interests have not been affected. But it has not shown that the accused!s security interests have not been affected. (stress, anxiety, stigmatization ... ) This prejudice has been suffered beyond the length of time that can be legitimately supported on the basis of limmited institutional resources.

I R. v. O'Connor (1996), 44 CR.

Facts Bishop was charged w/ four counts of sexual offences, two involving rape alleged to have occured over a three year period some 25 years before. Accused applied for a jud`l stay of proceedings based on non-disclosure of several items. Trial judge dismissed the application finding the failure to disclose was an oversight. Accused applied again for a stay of proceedings based on the fact that the Crown was still unable to guarantee to the accused that ful. disclosure had been made. Trial judge stayed proceedings. CA allowed the appeal and directed a new trial.

L'H.-D.

1nd!1 rts to trial fairness and the general reputation of the crim'l jus system are fimdamental concerns underlying both the c/l doctrine of abuse of process and the charter. Charter is concerned w/ the rts of the individual + w/ the integrity of the judl system.

Conducting a prosecution in a manner that contravenes the community's basic sense of decency and fair play and thereby calls into question the integrity of the system is also an affront of const!l magnitude to the rts of the individual accused. Individual who is subject to abuse of process should be entitled to present arguments under the Charter and to request a just and appropriate remedy from a ct of competent jurisdiction.

Remedies less drastic than a stay of proceedings are of course available under s.24(l) mi situations where the "clearest of cases" treshold is not met but where it is proved on a balance of probabilities that s. 7 has been violated.

A stay of proceedings is only appropriate in the "clearest of cases" where the prejudice to the accused!s rt to make full answer and defence cannot be remedied or where irreparable prejudice would be caused to the integrity of the judl system if the prosecution were continued.

Appeal dismissed. R. v.Potvin [1993] S.C.R. 880.

Facts Accused charged w/ crim!1 negligence causing death. Information was sworn on Sept. 15, 1988. He was released from custody on an undertaking. A series of lengthy delays... and the trial date was set for Dec. 3, 1990! Accused applied on that date for a stay of proceedings under 24(l) on the grounds that his right to trial w/in a reasonable time had been infiinged.

Dec. 24 1990 AG appealed against the stay. Appeal was heard ApI 24 1992 during which the issue of the appellate delay was raised. June 22 1992 CA allowed the appeal, set aside the stay and remitted the matter for trial on an expedited basis.

Held Appeal dismissed.

Analysis Sopinka J. (UH.-D., Gonthier, Cory, Iacobucci concurring)

To determine if the delay was unreasonable CA weighed the explanation for the delay and prejudice to the accused to detern3ine whether the appellants!s constitutional right to be tried w/in a reasonable time had been breached.

Although prejudice is only one factor to be taken into account in the balancing process, a conclusion that prejudice was absent or that it was minimal is significant.

Prejudice is consistent w/ an analysis of s. I lb) bc it is to protect the dual rights of the accused and the dual societal interest in seeing an accused treated humanely and fairly and in seeing those accused of crimes be brought to trial.

Weighing all the evidence, J.A. found that the delay was not unreasonable. He concluded that the prejudice inferred was minimal + respondent never sought an earlier date for any instance. "there may be a fine line bet. acceptance of the inevitability of the pace of proceedings and contentment w/ the speed of process".

Appellate Delay Issue was raised in the CA but was not dealt w/. S. I lb) does not apply to delay in respect of an appeal from the conviction by the accused nor an appeal from an acquittal by the Crown. Moreover there is no distinction bet. an acquittal after trial and a jud'I stay. Objects and purposes of s.11(b) (R v. Morin)

S. I I(b) only protects the interests against the consequences flowing from a formal charge. During the period of acquittal and the service of a notice of appeal, the person acquitted is not a person charged. Upon the appears being filed, there is a possibility that the acquittal will be set aside and the charge will be revived. In this respect the former accused is like the suspect against whom an investigation has been completed and charges are contemplated awaiting a decision by the prosecutor.

There is even less reason to extend the protection of s. I l(b) to a convicted person who appeals. The appeal itself is not gvtal action.

No need to distinguish bet. acquittal and jud'I stay; in both cases, the accused can plead autrefois acquit and no proceedings may be brought in respect of the same charge unless the acquittal or stay is set aside on appeal.

Crim!I appellant or respondent still has a remedy when delay proceedings affect the fairness of the trial. While s. I lb) does not apply, s.7 may in appropriate circumstances afford a remedy.

McLachfin J. (Lamer, Major concurring)

Rights protected by s. I I(b) are all engaged in the period bet. a verdict or a stay and the final disposition of the crim! I charge. The acquitted person having been found not guilty, faces the prospect of conviction and incarceration.

The narrow remedy proposed by Sopinka, would not provide a remedy capable of meeting these concerns. He concludes that s.7 would apply where the delay is so long that it constitutes abuse of process. Accused would be entitled to a remedy if he could show that a new trial would be so unfair as to amount to an abuse of the ct's process.

Doctrine of abuse of process should be applied only in the clearest of cases. This has been taken to impose a higher standard of proof than would face an accused relying on a breach of rights under the Charter.

A stay of proceedings is the min. and generally appropriate remedy. However, where a verdict has been entered a stay may not be appropriate. To release a convicted killer into society bc the appeal he /she chose to bring took more time than reasonable, would be to grant a remedy whichfar outstrips the wrong and which overlooks the important societal interest in security of our members of the public. Best is a flexible approach to remedy.

LaForest agrees w/ Sopinka that I l(b) does not apply to appellate delay ; and agrees w/ MacLachlin that a stay should not as often be used as the appropriate remedy for appellate delay as for trial delay. i Rourke v. R. [1978] 1 SCR 1021.

Facts Oct. 5th 1971alleged kidnapping an robbery was conitnitted. Feb. 26th 1973 an information was sworn against the appellant. Apl. 3rd 1973 a warrant was issued and the appellant was arrested. Jun. 1973 a preliminary inquiry was held and the applellant was committed for trial. Nov. 21st 1973 he appeared for trial and moved for a stay of proceedings as an abuse of process. Delay was prejudicial to his defence.

An application for mandamus was dismissed. This was reversed on appeal.

McIntyre JA An accused may suffer prejudice by delay. But such delay would ordinaily raise a substantive defence. Where a judge having a jurisdiction to exercise declines to do so bc of a decision on a preliminary question, which does not go to the merits as regards either fact or law, mandamus will lie. In thiscase, mandamus should go.

Pigeon J. (Mardand, Ritchie, Beetz, DeGranpr6 JJ concurring) There is no rule in crinil law that prosecutions must be instituted promptly and ought not to be permitted to be proceeded w/ if a delay in instituting them may have caused prejudice to the accused. There is no discretionary power in cts of critnl jurisdiction to stay proceedings regularly instituted bc the prosecution is considered oppressive.

Quotes Viscomt Dilhorne in Mills v. Cooper [1967] : A judge must keep out of the arena. He should not have or appear to have any responsibility for the institution of a prosecution.

Absence of any prov. in the Crim' I Code contemplating the staying of an indictment is a strong indication against the existence of any power to grant such stay.

Laskin J

Abuse of process has been put forward in a number of Can cases as a ground for staying or prohibiting crim' I proceedings where there has been excessive and unexplained delay *in bringing the accused to trial after the charge or after commital for trial.

Abuse of process as a ground for quashing charges or staying proceedings was invoked in cases where an accused was removed for trial to a place remote from that where the offences were allegedly committed and was prejudiced in his ability to bring witnesses, and thus denied his right to make full answer and defence.

There is a utility of a general principle of abuse of process which judges should be able to invoke in appropriate circumstances to mark their control of the process of their cts and to require fair behaviour of the Crown towards accused persons.

In the case at bar, the cts have no business telling the police that they did not proceed expeditiously enough for their investigation, and then impose a sanction of a stay when the prosecution is initiated. Prosecutions initiated a lenghty period after the alleged commission of an offence must be left to take its course and to be dealt w/ by the ct on the evidence, which judges are entitled to way for cogency as well as credibility. The ct can call for an explanation of any untoward delay and may be in a position accordingly to assess the weight of some evidence.

No basis upon which abuse of process could be invoked here and hence it did not provide an occasion for the exercise of discretion.

Appeal dismissed.